Eyelid Flashcards

1
Q
A 10-month-old boy is brought to the office because of the ocular defect shown in the photograph. Which of the following is the correct term for this anomaly? ( notch on medial upper lid)
A) Anophthalmia
B) Coloboma
C) Congenital cataract
D) Palpebral fissure
E) Tessier No. 6 cleft
A

B) Coloboma

How well did you know this?
1
Not at all
2
3
4
5
Perfectly
2
Q

Coloboma

A

Coloboma is a congenital ocular defect of the eyelid, iris, retina, choroid, or optic disk. The defects can range in size from a small notch to a large structural cleft.

How well did you know this?
1
Not at all
2
3
4
5
Perfectly
3
Q

Origin of colobomas

A

Palpebral colobomas are thought to arise from a localized growth disturbance, while colobomas of the iris, retina, and optic disk arise from defective closure of the optic fissure.

How well did you know this?
1
Not at all
2
3
4
5
Perfectly
4
Q

Vision obstruction and colhbomas

A

Upper eyelid coloboma rarely affects vision; large defects of the lower eyelid can lead to corneal ulceration.

How well did you know this?
1
Not at all
2
3
4
5
Perfectly
5
Q

Anopthalmia

A

Total absence of the eye

How well did you know this?
1
Not at all
2
3
4
5
Perfectly
6
Q

Congenital cataract

A

A congenital cataract is a lens opacity that is present in 1:4000 to 1:10,000 newborns.

How well did you know this?
1
Not at all
2
3
4
5
Perfectly
7
Q

Palpebral fissure

A

The palpebral fissure is the natural opening between the upper and lower eyelids.

How well did you know this?
1
Not at all
2
3
4
5
Perfectly
8
Q

Tessier No. 6 cleft

A

A Tessier No. 6 cleft involves the inferolateral aspect of the lower eyelid, inferior orbital rim, and the zygoma. This cleft often has an associated lower eyelid coloboma and is related to Treacher Collins syndrome.

How well did you know this?
1
Not at all
2
3
4
5
Perfectly
9
Q
A 30-year-old man comes to the office because of excessive tearing of the left eye 3 months after repair of a deep laceration of the left medial canthus. Physical examination confirms epiphora of the left eye. A missed injury to the lacrimal drainage system is suspected. Which of the following is most appropriate to evaluate the suspected injury?
A) CT scan
B) Jones test
C) MRI
D) Nasal endoscopy
E) Schirmer test
A

B) Jones test

How well did you know this?
1
Not at all
2
3
4
5
Perfectly
10
Q

Jones Test

A

The Jones test evaluates the drainage component of the lacrimal system. There are two steps in this test. The first step involves instilling two drops of a 2% fluorescein solution into the conjunctiva and observing the appearance or absence of this dye in the ipsilateral middle turbinate by means of a curved, cotton-tipped applicator. When dye is noted on the applicator within 3 minutes, the drainage system is intact. Therefore, if no obstruction is noted, the epiphora is caused by hypersecretion. If no dye passes through the nasolacrimal duct, then a secondary dye test is performed by force-injecting 1 mL of saline through the punctum. If this is done and the dye appears in the nose, then a functional or incomplete block of the nasolacrimal duct is present; if no dye appears, then a complete block is diagnosed.

How well did you know this?
1
Not at all
2
3
4
5
Perfectly
11
Q

Differentiating a canalicular block from a nasolacrimal duct block

A

Differentiating a canalicular block from a nasolacrimal duct block is done by cannulating and injecting 1 mL of saline through a canaliculus and observing the passage of clear fluid out the other canaliculus. A No. 00 Bowman probe can also be passed through the canaliculus, and the distance to the obstruction can be measured.

How well did you know this?
1
Not at all
2
3
4
5
Perfectly
12
Q

A No. 00 Bowman probe can be passed through the canaliculus to measure the distance to a canalicular obstruction. Typical distances:

A

A No. 00 Bowman probe can be passed through the canaliculus to measure the distance to an obstruction: Typical distances are 8 mm to common canaliculus, 10 to 12 mm to the tear sac, and 16 mm to the upper end of the nasolacrimal duct.

How well did you know this?
1
Not at all
2
3
4
5
Perfectly
13
Q

Nasal endoscopy role in evaluating a canalicular obstruction

A

Nasal endoscopy can only examine the nasal cavity for an abnormal location of the meatus under the middle turbinate. It can also determine if there is a physical obstruction of the lower end of the nasolacrimal duct at that location because of polyps or granulation tissue. This test could be indicated if the Jones test were to show partial obstruction.

How well did you know this?
1
Not at all
2
3
4
5
Perfectly
14
Q

Schirmer test

A

The Schirmer test is most useful in diagnosing lacrimal hyposecretion.

How well did you know this?
1
Not at all
2
3
4
5
Perfectly
15
Q

Epiphora

A

Overflow of tears on the face

How well did you know this?
1
Not at all
2
3
4
5
Perfectly
16
Q
An 18-year-old man comes to the emergency department 2 hours after beingpunched in the right eye during a fistfight. Physical examination shows swelling, diplopia, and a significant limitation of downward gaze in the affected eye. He is able to rotate the eye in all other directions. This finding is most consistent with dysfunction of which of the following extra ocular muscles?
A) Inferior oblique
B) Inferior rectus
C) Superior oblique
D) Superior rectus
A

B) Inferior rectus

How well did you know this?
1
Not at all
2
3
4
5
Perfectly
17
Q

The inferior rectus is innervated by:

A

CN III

How well did you know this?
1
Not at all
2
3
4
5
Perfectly
18
Q

The inferior oblique is innervated by:

A

CN III

responsible for upward and outward rotation of the eye.

How well did you know this?
1
Not at all
2
3
4
5
Perfectly
19
Q

The superior oblique is innervated by:

A

The superior oblique is innervated by the trochlear nerve(cranial nerve IV) and is responsible for downward and outward rotation of the eye.

How well did you know this?
1
Not at all
2
3
4
5
Perfectly
20
Q

The superior rectus is innervated by:

A

CN III

How well did you know this?
1
Not at all
2
3
4
5
Perfectly
21
Q
A 23-year-old man comes to the office for evaluation of unilateral blepharoptosis. On examination, the excursion of the eyelid margin is measured from downgaze to upgaze while the eyebrow is manually fixed against the supraorbital rim. Which of the following would best approximate the normal excursion distance of levator function for this patient?
A) 2 to 6 mm
B) 7 to 11 mm
C) 12 to 16 mm
D) 17 to 21 mm
E) 21 to 25 mm
A

C) 12 to 16 mm

How well did you know this?
1
Not at all
2
3
4
5
Perfectly
22
Q

Normal excursion of elevator function

A

12 to 16 mm

How well did you know this?
1
Not at all
2
3
4
5
Perfectly
23
Q

A 46-year-old woman comes to the office for consultation about improving the appearance of her “saggy” upper eyelids. Physical examination shows moderate skin redundancy in both upper eyelids. The ciliary margin of the upper eyelid is located 1 mm below the superior limbus on the right and 3 mm below the superior limbus on the left. Levator excursion is 14 mm bilaterally. In addition to excision of excess skin bilaterally, which of the following is the most appropriate treatment?
A) Brow lift surgery on the left
B) Frontalis suspension in the left eye
C) Levator aponeurosis plication in the left eye
D) Orbicularis plication in the left eye

A

C) Levator aponeurosis plication in the left eye

The upper eyelid skin redundancy can be addressed with a standard upper eyelid blepharoplasty with skin excision and, if needed, a slip of orbicularis muscle. The mild ptosis of her left upper eyelid will be accentuated after the blepharoplasty if left untreated. Since her levator function is excellent, her ptosis can be corrected with a simple plication of the distal levator aponeurosis. This can be accomplished through an upper blepharoplasty incision.

How well did you know this?
1
Not at all
2
3
4
5
Perfectly
24
Q

Limbus

A

The corneal limbus is the border of the cornea (the transparent front part of the eye that covers the iris, pupil, and anterior chamber) and the sclera (the white of the eye)

How well did you know this?
1
Not at all
2
3
4
5
Perfectly
25
Q

Normal anatomical position of the ciliary margin of the upper eyelid

A

The normal anatomical position of the ciliary margin of the upper eyelid is 1 mm below the upper corneoscleral junction in straight gaze.

How well did you know this?
1
Not at all
2
3
4
5
Perfectly
26
Q

Landmark for ptosis measurement

A

The upper limbus: the upper corneoscleral junction is the landmark for ptosis measurement

How well did you know this?
1
Not at all
2
3
4
5
Perfectly
27
Q

Correction of severe ptosis/poor elevator function

A

Patients with severe ptosis (3 mm or more) usually have poor levator function, therefore frontalis suspension is required.

How well did you know this?
1
Not at all
2
3
4
5
Perfectly
28
Q

Correction of moderate ptosis/moderate elevator function

A

In patients with moderate function (6 to 10 mm), levator advancement and resection is required.

How well did you know this?
1
Not at all
2
3
4
5
Perfectly
29
Q

Correction of mild ptosis / excellent elevator function

A

For patients with excellentlevator function (10 mm or more) and mild ptosis, aponeurotic surgery (plication) is appropriate.

How well did you know this?
1
Not at all
2
3
4
5
Perfectly
30
Q

How much will the eyelid be elevated per amount of elevator advancement?

A

In levator advancement surgery, the eyelid will be elevated approximately 1 mm for every 3 mm of advancement.

How well did you know this?
1
Not at all
2
3
4
5
Perfectly
31
Q

Fasanella-Servat procedure

A

In a Fasanella-Servat procedure, a portion of the posterior lamella of the eyelid is resected to improve mild eyelid ptosis by shortening the levator muscle. This procedure would not be used in a patient who requires a skin resection since it uses a posterior approach.

How well did you know this?
1
Not at all
2
3
4
5
Perfectly
32
Q
A 68-year-old woman is scheduled to undergo a bilateral blepharoplasty with unilateral repair of a levator aponeurosis dehiscence. During the dissection, the distal end of the dehisced levator muscle is reattached to the tarsal plate. Which of the following anatomic structures is most likely to be visualized deep to the repair?
A ) Capsulopalpebral fascia
B ) Conjunctiva
C ) Müller muscle
D ) Orbital septum
E ) Whitnall ligamen
A

C ) Müller muscle

The Müller muscle lies below the levator insertion to the superior border of the tarsus and is visualized during the levator muscle repair
.The conjunctiva is located deep to the Müller muscle and should not be visualized during the muscle reinsertion to the tarsus. The orbital septum and retro-orbicularis oculi are anterior and superior to the levator tendon. The Whitnall ligament is also superior to the levator tendon.

How well did you know this?
1
Not at all
2
3
4
5
Perfectly
33
Q

During a dissection, the distal end of the dehisced levator muscle is reattached to the tarsal plate. What should be visualized deep to the repair?

A

the Muller muscle

How well did you know this?
1
Not at all
2
3
4
5
Perfectly
34
Q

What are the layers of the eyelid?

A

The layers of the eyelid are the conjunctiva, Müller muscle, levator muscle, orbital fat, orbital septum, retro-orbicularis oculi fat, orbicularis oculi muscle, and skin.

How well did you know this?
1
Not at all
2
3
4
5
Perfectly
35
Q

Where is the Muller muscle vs the levator muscle?

A

deep

How well did you know this?
1
Not at all
2
3
4
5
Perfectly
36
Q

Where is the conjunctiva vs the levator muscle?

A

The conjunctiva is deep to the Muller muscle, which is deep to the levator

How well did you know this?
1
Not at all
2
3
4
5
Perfectly
37
Q

Where is the Whitnall ligament vs the levator muscle?

A

superior

How well did you know this?
1
Not at all
2
3
4
5
Perfectly
38
Q

Capsulopalpebral fascia

A

The capsulopalpebral fascia is the extension of the lower eyelid retractors and fuses at the inferior aspect of the lower eyelid tarsus

How well did you know this?
1
Not at all
2
3
4
5
Perfectly
39
Q

A 40-year-old woman is scheduled to undergo surgical correction ofbilateral upper eyelid dermatochalasis and mild blepharoptosis. A Fasanella-Servat procedure is planned. Which of the following is the most likely disadvantage of performing this procedure instead of a levator aponeurosis reinsertion?
A ) Dry eyes
B ) Eyelid asymmetry
C ) Incomplete correction of eyelid ptosis
D ) No removal of excess eyelid fold skin
E ) Overcorrection of eyelid position

A

D ) No removal of excess eyelid fold skin

One of the major disadvantages of the Fasanella-Servat procedure is that it does not address excess skin of the eyelid fold. Dry eyes, incompletecorrection, overcorrection, and eyelid asymmetry are possible complications of both procedures.

How well did you know this?
1
Not at all
2
3
4
5
Perfectly
40
Q

What is the major disadvantage of the Fasanella-Servat procedure vs a elevator aponeurosis reinsertion?

A

The Fasanella-Servat does not address excess skin of the eyelid fold.

How well did you know this?
1
Not at all
2
3
4
5
Perfectly
41
Q

A 64-year-old man is evaluated for reconstruction of a defect of the lower eyelid following resection of a 1.4-cm nodular basal cell carcinoma. Examination shows an 80% full-thickness defect of the lateral lower eyelid. Which of the following is the most appropriate method of reconstruction?
A ) Cantholysis, lateral canthotomy, and primary closure
B ) Cheek advancement flap with composite graft for lining
C ) Composite graft from the ear
D ) Forehead flap with septal cartilage grafting
E ) Hughes tarsoconjunctival flap with skin grafting

A

B ) Cheek advancement flap with composite graft for lining

The most appropriate method of reconstruction is the cheek advancement flap with a nasal septal cartilage and lining graft for internal lining.

How well did you know this?
1
Not at all
2
3
4
5
Perfectly
42
Q

Reconstruction of major (>75%) defect of the lower eyelid

A

The most appropriate method of reconstruction is the cheek advancement flap with a nasal septal cartilage and lining graft for internal lining. The cheek advancement flap can be elevated widely and rotated without tension to provide anterior coverage of the defect, while the composite graft is used for lining and support of the lower eyelid.

How well did you know this?
1
Not at all
2
3
4
5
Perfectly
43
Q

Reconstruction of lower lid defects <50%

A

Cantholysis, lateral canthotomy, and primary closure are most useful for defects that are less than 50%. Similarly, theHughes tarsoconjunctival flap is best used in defects that are less than 50%, as larger flaps would result in significant deformity of the upper eyelid.

How well did you know this?
1
Not at all
2
3
4
5
Perfectly
44
Q

Composite grafts from the ear and lower eyelid reconstruction

A

Composite grafts from the ear are rarely used for lower eyelid reconstruction because the tissues are usually thicker than the lower eyelid and may be associated with partial or complete graft loss.

How well did you know this?
1
Not at all
2
3
4
5
Perfectly
45
Q

A 69-year-old woman comes to theoffice because she has had dryness, tearing, and irritation in both eyes for the past two years. Physical examination shows outward turning of the lower eyelid margin. Anterior eyelid distraction is 7 mm from the globe. Snap-back test is greater than one second. Muscle tone is normal bilaterally. Which of the following is the most likely cause of lower eyelid ectropion in this patient?
(A)Dehiscence of lower eyelid retractors
(B)Horizontal laxity of the lower eyelid
(C)Loss of lower eyelid tone secondary to paralysis of orbicularis oculi muscle
(D)Neoplasia within the lower eyelid causing the eyelid to be pulled away from the globe
(E)Vertical shortening of the anterior lamella of the eyelid

A

(B)Horizontal laxity of the lower eyelid

How well did you know this?
1
Not at all
2
3
4
5
Perfectly
46
Q

Physical findings suggesting abnormal horizontal lid laxity

A

Physical examination findings of abnormal snap-back test and distraction test in the patient described suggest abnormal horizontal lid laxity

How well did you know this?
1
Not at all
2
3
4
5
Perfectly
47
Q

Causal factors leading to ectropion

A

Causal factors leading to ectropion include horizontal laxity of the eyelid (involutional), vertical shortening of the anterior lamella of the eyelid (congenital or cicatricial), paralysis of the orbicularis oculi muscle (secondary to paralytic entropion) causing loss of eyelid muscular tone, and neoplasia within the lower eyelid pulling or forcing the eyelid away from the globe.

How well did you know this?
1
Not at all
2
3
4
5
Perfectly
48
Q

An 82-year-old woman comes to the office because she has had excessive tearing and irritation of both eyes for the past three months. Physical examination shows severe lower eyelid laxity, scleral show, downward drift of the lateral canthus, and a shortened intercanthal distance bilaterally. Entropion is suspected. Which of the following is most likely to confirm the suspected diagnosis?
(A)Animation test of the orbicularis oculi muscle
(B)Hertel exophthalmometry
(C)Jones dye test
(D)Lower eyelid snap-back test
(E)Schirmer test

A

(A)Animation test of the orbicularis oculi muscle

How well did you know this?
1
Not at all
2
3
4
5
Perfectly
49
Q

Involutional entropion can be confused with involutional ectropion because:

They are completely distinguishable when:

A

Involutional entropion can be confused with involutional ectropion in the static state because of the associated lower eyelid laxity in both instances.

The two conditions are completely distinguishable on animation of the orbicularis oculi muscle.

How well did you know this?
1
Not at all
2
3
4
5
Perfectly
50
Q

What happens on attempted eyelid closure in cases of involutional entropion?

A

Inversion of the lower eyelid occurs on attempted eyelid closure in cases of involutional entropion.

How well did you know this?
1
Not at all
2
3
4
5
Perfectly
51
Q

Causes of involutional entropion

A

Causal factors include orbicularis dysfunction with the preseptal portion overriding the pretarsal portion, upper eyelid “closure kick,” disinsertion of lower eyelid retractors, loss of horizontal and vertical eyelid support, and loss of orbital fat volume

How well did you know this?
1
Not at all
2
3
4
5
Perfectly
52
Q

A 32-year-old woman comes to the office for follow-up examination six months after she underwent repair of a fracture of the left orbit. A preoperative photograph is shown. Physical examination shows cicatricial ectropion of the lower eyelid. Which of the following is the most appropriate management? (lower lid lacks height)
(A)Complete tarsorrhaphy
(B)Pentagonal wedge resection of the lower eyelid
(C)Placement of a graft to the lower eyelid
(D)Punctal occlusion ofthe lower eyelid
(E)Resuspension of the lower eyelid retractors

A

(C)Placement of a graft to the lower eyelid

Protection of the ocular surfaces is the primary goal in managing the cicatricial ectropion of the lower eyelid in the patient described.The patient’s lower eyelid lacks height. Scarring between the capsulopalpebral fascia (lower eyelid retractors) and the orbital septum because of trauma or following aggressive lower eyelid blepharoplasty can cause vertical contracture of the lower eyelid, resulting in corneal exposure. Lower eyelid position can only be restored by placement of an autogenous or alloplastic spacer graft (hard palate, donor sclera, auricular cartilage, tarsus, and others), as in the postoperative image shown. In conjunction,eyelid-tightening procedures such as lateral canthopexy and canthoplasty address horizontal skin laxity

How well did you know this?
1
Not at all
2
3
4
5
Perfectly
53
Q

Primary goal in managing cicatricial ectropion w/ lower lid which lacks height

A

Protection of the ocular surfaces is the primary goal in managing the cicatricial ectropion of the lower eyelid in the patient described.

How well did you know this?
1
Not at all
2
3
4
5
Perfectly
54
Q

What can cause vertical contracture of the lower eyelid?

A

Scarring between the capsulopalpebral fascia (lower eyelid retractors) and the orbital septum because of trauma or following aggressive lower eyelid blepharoplasty can cause vertical contracture of the lower eyelid, resulting in corneal exposure.

How well did you know this?
1
Not at all
2
3
4
5
Perfectly
55
Q

Most likely diagnosis: A 66-year-old man with visual obstruction of the left eye and elevation of the supratarsal crease

A

Senile ptosis

How well did you know this?
1
Not at all
2
3
4
5
Perfectly
56
Q

Most likely diagnosis: A 60-year-old woman with general muscle fatigue and bilateral upper visual field obstruction that worsens at the end of the day

A

Myasthenia gravis

How well did you know this?
1
Not at all
2
3
4
5
Perfectly
57
Q

A 26-year-old woman comes to the office after being referred by an ophthalmologist because of scleral show of the left lower eyelid. Three weeks ago, she sustained a laceration of the lower eyelid, which was sutured in the emergency department. Which of the following is the most appropriate next step in management?
(A)Tear supplementation and massage of the scar
(B)Injection of a corticosteroid into the scar and silicone sheeting
(C)Lysis of scar adhesions and septal cartilage interposition grafting
(D)Excision of the scar and full-thickness skin grafting
(E)Lateral canthopexy and cheek advancement flap

A

(A)Tear supplementation and massage of the scar

At three weeks into the postoperative period, the wounds are still in the active phase of healing. Over the next six to nine months, as the collagen with the scar remodels, the lid may return to a more normal position. Therefore, during this time frame, the ectropion should be managed with scar tissue massage and a regimento prevent complications related to dry eyes, such as tear supplementation and eye patching. Surgical intervention is warranted if there is no improvement after this time frame or if conservative therapy is ineffective and there is a risk of visual compromise.

Injection of a corticosteroid can reduce inflammation and soften the scar, allowing the eyelid to return to a better position. However, use of corticosteroids in the lower eyelid is not advocated because of the risks of tissue atrophy and skin discoloration

How well did you know this?
1
Not at all
2
3
4
5
Perfectly
58
Q

Ectropion is classified as:

A

In general, ectropion is classified as involutional, cicatricial, or neurogenic.

How well did you know this?
1
Not at all
2
3
4
5
Perfectly
59
Q

Assessing for midlamellar scar

A

Placing upward traction on the eyelid can identify scar contracture in this location. If adequate skin is present, the lower eyelid should reach at least the level of the midpupil. If this movement is restricted, midlamellar scarring is likely.

How well did you know this?
1
Not at all
2
3
4
5
Perfectly
60
Q

Treatment of midlamellar scar

A

Lysis of scar adhesions and the placement of an interpositional graft are indicated for cases of midlamellar scar.

How well did you know this?
1
Not at all
2
3
4
5
Perfectly
61
Q

A 24-year-old man who works as a chef comes to the office because he has ectropion of the left lower eyelid and ulceration of the cornea five months after he sustained an oil burn to the left side of the face. On physical examination, the burn scars are hypertrophic and immature, extending from the infraorbital rim to the mandibular border. Which of the following is the most effective management of the ectropion?
(A) Full-thickness skin grafting
(B) Massage of the eyelid and injection of a corticosteroid
(C) Palatal mucosal grafting
(D) Tarsoconjunctival flap
(E) Tissue expansion

A

(A) Full-thickness skin grafting

How well did you know this?
1
Not at all
2
3
4
5
Perfectly
62
Q

Cicatricial ectropion and burns

A

Cicatricial ectropion can be prevented by early surgical intervention with burn excision and grafting, thereby avoiding the need for late reconstructive procedures.

How well did you know this?
1
Not at all
2
3
4
5
Perfectly
63
Q

Cicatricial ectropion treatment

A

Cicatricial ectropion is best treated by full-thickness skin grafting to replace the skin deficit. A tarsal support procedure, such as a lateral canthoplasty, may be needed to fully restore the tone and position of the eyelid. Depending on their thickness and distribution, scars may be released by incision or excision.

How well did you know this?
1
Not at all
2
3
4
5
Perfectly
64
Q

When should burn reconstruction be performed?

A

Most burn reconstruction should be delayed until scars have matured sufficiently, usually by one year after injury. Earlier intervention is needed when a vital skin function is impaired, such as corneal protection.

How well did you know this?
1
Not at all
2
3
4
5
Perfectly
65
Q

Scarred skin and tissue expansion

A

Scarred tissue does not expand effectively, limiting the role of tissue expansion in burned/scarred patients

How well did you know this?
1
Not at all
2
3
4
5
Perfectly
66
Q

Which of the following best differentiates the Asian upper eyelid from the Occidental upper eyelid?
(A) Absence of epicanthal folds
(B) Decreased amount of suborbicularis oculi fat
(C) More superior fusion of the orbital septum and levator aponeurosis
(D) Relative lack of insertions from the levator aponeurosis into the dermis
(E) Well-defined supratarsal lid fold with a larger pretarsal segment

A

(D) Relative lack of insertions from the levator aponeurosis into the dermis

How well did you know this?
1
Not at all
2
3
4
5
Perfectly
67
Q

Cause of lack of supra tarsal fold in Asians

A

In as much as 50% of the Asian population, there is a general lack of insertion of the levator aponeurosis into the dermis, causing a lack of a supratarsal fold.

How well did you know this?
1
Not at all
2
3
4
5
Perfectly
68
Q

Fusion of orbital septum to the elevator aponeurosis in asians

A

The fusion of the orbital septum to the levator aponeurosis is typically more caudad and decreases the width of the pretarsal segment of the supratarsal lid fold when it is present.

How well did you know this?
1
Not at all
2
3
4
5
Perfectly
69
Q

Periorbital fat in asians

A

There are generally increased amounts of retro-orbicularis oculi fat and suborbicularis oculi fat. T

How well did you know this?
1
Not at all
2
3
4
5
Perfectly
70
Q

Differences between Asian and Occidental eyelids

A

The Asian eyelid has specific anatomic variations compared with the Occidental eyelid. First, in as much as 50% of the Asian population, there is a general lack of insertion of the levator aponeurosis into the dermis, causing a lack of a supratarsal fold. The fusion of the orbital septum to the levator aponeurosis is typically more caudad and decreases the width of the pretarsal segment of the supratarsal lid fold when it is present. There are generally increased amounts of retro-orbicularis oculi fat and suborbicularis oculi fat. The Asian eyelid is likely to have more epicanthal folds than is the Occidental eyelid.

How well did you know this?
1
Not at all
2
3
4
5
Perfectly
71
Q

A 65-year-old man has a 3-cm-diameter open wound of the medial cheek inferior to the lower eyelid after undergoing Mohs’ micrographic surgery for excision of nodular basal cell carcinoma. Snap-back test of the lower eyelid shows poor tone. Reconstruction of the defect is performed with a cervicofacial flap. Which of the following is the most appropriate next step in management to avoid a deformity of the lower eyelid?
(A) Application of adhesive bandages to the lower eyelid and daily massage
(B) Placementof a temporary tarsorrhaphy (Frost) suture
(C) Reconstruction with a tarsoconjunctival flap
(D) Horizontal shortening of the lower eyelid and lateral canthopexy
(E) Full-thickness skin grafting of the lower eyelid

A

(D) Horizontal shortening of the lower eyelid and lateral canthopexy

How well did you know this?
1
Not at all
2
3
4
5
Perfectly
72
Q

The snapback test assesses:

A

The snap-back test can be used to assess horizontal laxity of the eyelid.

How well did you know this?
1
Not at all
2
3
4
5
Perfectly
73
Q

Cicatricial changes

A

Due to loss of skin

How well did you know this?
1
Not at all
2
3
4
5
Perfectly
74
Q

Use of tarsorrhaphy to protect the cornea

A

The use of tarsorrhaphy is a temporary means to protect the cornea from exposure. The suture itself will not prevent lower eyelid malposition.

How well did you know this?
1
Not at all
2
3
4
5
Perfectly
75
Q

A 3-year-old boy is brought to the office by his parents because of new onset of bilateral epiphora. The boy’s parents say that similar symptoms occurred in one of their older children but resolved without treatment. On physical examination, the lashes of both lower eyelids rub against the inferior cornea. Which of the following is the pathophysiologic mechanism underlying this patient’s condition?
(A) Abnormal attachment of the canthal tendons
(B) Abnormal attachment of the orbital septum
(C) Enophthalmos
(D) Laxity of the tarsal plate
(E) Redundancy of skin and orbicularis muscle

A

(E) Redundancy of skin and orbicularis muscle

How well did you know this?
1
Not at all
2
3
4
5
Perfectly
76
Q

A 13-month-old girl has had tearing and discharge from the right eye since birth. Which of the following is the most appropriate management?
(A) Observation
(B) Instruction of the parents in massage with antibiotic ointment
(C) Silastic intubation
(D) Probing of the nasolacrimal duct
(E) Dacryocystorhinostomy

A

(D) Probing of the nasolacrimal duct

How well did you know this?
1
Not at all
2
3
4
5
Perfectly
77
Q

Congenital tearing problem is most likely due to..

A

A child with congenital tearing is likely to have a nasolacrimal duct problem. Punctual agenesis, lacrimal sac fistula, and other rare abnormalities should be ruled out with dye disappearance testing, which usually is markedly asymmetric in a nasolacrimal duct problem.

How well did you know this?
1
Not at all
2
3
4
5
Perfectly
78
Q

Management of congenital nasolacrimal duct problems

A

Generally, a nasolacrimal duct problem should be treated with massage and antibiotic drops until the child is age 12 to 13 months. For about 70% of children with tearing at age 6 months, this conservative treatment leads to resolution by age 12 months. If tearing persists, probing of the nasolacrimal duct should be performed. The longer probing is delayed beyond age 13 months, the greater the number and complexity of the procedures needed to successfully treat congenital dacryostenosis.

How well did you know this?
1
Not at all
2
3
4
5
Perfectly
79
Q

When should congenital nasolacrimal duct problems be probed?

A

If tearing persists beyond 12 months, probing of the nasolacrimal duct should be performed. The longer probing is delayed beyond age 13 months, the greater the number and complexity of the procedures needed to successfully treat congenital dacryostenosis. Therefore, initial probing and irrigation should be performed before age 13 months.

How well did you know this?
1
Not at all
2
3
4
5
Perfectly
80
Q

If probing is not successful for congenital nasolacrimal duct problem…

A

Probing should be performed between 12 and 13 months (after conservative management w/ antibiotic ointment and massage, and before greater risk of requiring more numerous and complex procedures.)

If probing is unsuccessful, Silastic intubation should be done.

How well did you know this?
1
Not at all
2
3
4
5
Perfectly
81
Q

Dacryocystorhinostomy and congenital nasolacrimal duct problem

A

Probing should be performed between 12 and 13 months (after conservative management w/ antibiotic ointment and massage, and before greater risk of requiring more numerous and complex procedures.)

If probing is unsuccessful, Silastic intubation should be done.

Dacryocystorhinostomy is reserved for those rare cases that do not respond to Silastic intubation.

How well did you know this?
1
Not at all
2
3
4
5
Perfectly
82
Q

A 16-year-old girl has persistent ptosis of the eyelid (shown) six months after undergoing reconstruction of the forehead and supraorbital bar for fibrous dysplasia. One month postoperatively, a wound developed from extrusion of hardware through the medial eyelid. Debridement of the wound with excision of the involved inflammatory tissue and reconstruction with a pericranial flap were performed at that time. On current examination, no levator function is noted medially in the eyelid and lateral movement of the eyelid is minimal. Which of the following interventions is the most appropriate next step in management? (the function as shown in the photographs is minimal)
(A) Lysis of adhesions
(B)Kuhnt-Szymanowski procedure
(C) Fasanella-Servat procedure
(D) Suspension to the frontalis muscle with fascia lata grafting
(E) Advancement of the levator muscle

A

(D) Suspension to the frontalis muscle with fascia lata grafting

This patient has traumatic ptosis as a complication of prior surgical procedures. At the time of debridement, either a portion of the levator muscle was excised or the muscle function is limited by scar tissue. In either case, the function as shown in the photographs is minimal, and the ptosis is best managed by a frontalis suspension procedure.

How well did you know this?
1
Not at all
2
3
4
5
Perfectly
83
Q

Choice of sling for frontal suspension procedure for ptosis

A

fascia lata

How well did you know this?
1
Not at all
2
3
4
5
Perfectly
84
Q

The Fasanella-Servat procedure

A

The Fasanella-Servat procedure involves excision of a portion of the conjunctiva, tarsus, orbital septum, levator aponeurosis, and Müller muscle. It can be used in cases of mild ptosis (1-2 mm). Levator function must be present.

How well did you know this?
1
Not at all
2
3
4
5
Perfectly
85
Q

The Kuhnt-Szymanowski procedure

A

The Kuhnt-Szymanowski procedure is used to correct lower eyelid ectropion. It involves a wedge excision of the lower eyelid.

How well did you know this?
1
Not at all
2
3
4
5
Perfectly
86
Q
Hyphema results from traumatic hemorrhage of which of the following ocular structures? 
(A) Anterior chamber
(B) Conjunctiva
(C) Lens
(D) Posterior chamber
(E) Vitreous chamber
A

(A) Anterior chamber

How well did you know this?
1
Not at all
2
3
4
5
Perfectly
87
Q

Hyphema

A

Hyphema is traumatic hemorrhage of the anterior chamber of the eye, typically resulting from blunt trauma to ocular structures.

In patients with hyphema, vessels are torn in the iris or in the ciliary body, leading to onset of hemorrhage. The blood collects in the most inferior section of the anterior chamber, obscuring the lower portion of the iris

How well did you know this?
1
Not at all
2
3
4
5
Perfectly
88
Q

Anterior chamber anatomy

A

The anterior chamber of the eye is bordered by the cornea anteriorly and the iris and central portion of the lens posteriorly. This structure is filled with aqueous humor originating from the ciliary processes in the posterior chamber and flowing through the pupil into the anterior chamber.

How well did you know this?
1
Not at all
2
3
4
5
Perfectly
89
Q

Adverse consequences of hyphema

A

Significant hyphema may result in increased ocular pressure and/or permanent staining of the cornea. Because of the potential for these complications, screening is recommended in patients with facial trauma, and ophthalmologic referral is indicated in any patient with positive findings.

How well did you know this?
1
Not at all
2
3
4
5
Perfectly
90
Q

Treatment of hyphen

A

Treatment involves administration of acetazolamide and corticosteroid eye drops to decrease ocular tension.

How well did you know this?
1
Not at all
2
3
4
5
Perfectly
91
Q

Subconjunctival hemorrhage

A

Bleeding that occurs in the bulbar or palpebral conjunctiva is referred to as subconjunctival hemorrhage. This condition is seen in patients with facial trauma and results from extravasation of conjunctival capillaries. It also occurs in association with zygomatic fractures that extend through the lateral orbital wall in which there is bleeding along the side of the orbit and into the subconjunctival interstitium. Treatment of the fracture is likely to resolve the hemorrhage.

How well did you know this?
1
Not at all
2
3
4
5
Perfectly
92
Q

Treatment of subconjunctival hemorrhage

A

Due to facial fractures - Treatment of the fracture is likely to resolve the hemorrhage.

How well did you know this?
1
Not at all
2
3
4
5
Perfectly
93
Q

Blood in the posterior chamber of the eye

A

The posterior chamber is located behind the iris and anterior to the suspensory ligament; it appears as a halo around the lens. Although this chamber adjoins the ciliary processes, which may hemorrhage, the blood is not retained within it, but instead flows into the anterior chamber.

94
Q

Vitreous chamber

A

The vitreous chamber is bordered by the lens and suspensory ligaments anteriorly and the retina posteriorly. It contains a dense, jelly-like fluid that maintains slight pressure and provides even contact of the retina against the choroid.

95
Q
The common canaliculus enters the lacrimal sac at a point posterior to which of the following structures?
(A) Deep head of the preseptal muscle
(B) Deep head of the pretarsal muscle
(C) Lacrimal crest
(D) Medial canthal tendon
(E) Medial horn of the levator muscle
A

(D) Medial canthal tendon

96
Q

The common canaliculus enters the lacrimal sac at a point posterior to:

A

The common canaliculus enters the lacrimal sac at a point posterior to the medial canthal tendon.

97
Q

Anatomy of he medial canthal tendon

A

The medial canthal tendon is formed from the superficial heads of the pretarsal muscles and originates anterior to and above the lacrimal crest.

98
Q

Anatomy of the anterior and posterior lacrimal crests

A

The anterior and posterior lacrimal crests border the lacrimal fossa, which contains the lacrimal sac. The crests lie posterior to the canalicular entrance to the sac.

99
Q

Anatomy of the deep heads of the pretarsal muscles

A

The deep heads of the pretarsal muscles extend posterior to the lacrimal sac and join with the diaphragm of the scar to originate immediately behind the posterior lacrimal crest.

100
Q

Anaomy of the deep head of the preseptal muscle

A

The deep head of the preseptal muscle originates from the posterior lacrimal crest, just above the deep heads of the pretarsal muscles

101
Q
A 25-year-old man has excessive tear secretion and a chronic mucocele of the lacrimal sac. Jones I dye testing shows no dye, while Jones II dye testing shows dye within the tear sac. Dilatation of the puncta with probing and irrigation does not restore the patency of the lacrimal system. In order to restore nasolacrimal drainage in this patient, which of the following is the most appropriate operative procedure?
(A) Insertion of a Jones tube
(B) Canaliculodacryocystorhinostomy
(C) Conjunctivodacryocystostomy
(D) Conjunctivorhinostomy
(E) Dacryocystorhinostomy
A

(E) Dacryocystorhinostomy

102
Q

Management of a patient with positive Jones II dye test

A

In patients with positive Jones II dye tests, dilation of the puncta should then be performed; any fluid that passes into the nose indicates that the obstruction of the nasolacrimal duct has been cleared, and further probing is unnecessary.

103
Q

Jones II dye test: when dye-stained fluid is found at the inferior turbinate:

A

With this test, the nasolacrimal system is irrigated with 1 mL of saline via an irrigation cannula.

When dye-stained fluid is found at the inferior turbinate, there is partial obstruction of the lower canalicular system, most likely at the nasolacrimal duct.

104
Q

Jones II dye test: when dye-stained fluid is found within the tear sac

A

With this test, the nasolacrimal system is irrigated with 1 mL of saline via an irrigation cannula.

When dye-stained fluid is found within the tear sac, obstruction of the nasolacrimal duct can be diagnosed; the canaliculus and lacrimal pump are unaffected.

105
Q

Jones II dye test: when dye-stained fluid is found in the nose

A

ith this test, the nasolacrimal system is irrigated with 1 mL of saline via an irrigation cannula.

When dye-stained fluid is found in the nose (a negative test), the obstruction is most likely at the canalicular level.

106
Q

Negative findings of the Jones I test are indicative of:

A

Negative findings on the Jones I test are indicative of functional obstruction; if this occurs, the Jones II dye test should be performed immediately.

107
Q

Next step in patients with a positive Jones II dye test

A

In patients with positive Jones II dye tests, dilation of the puncta should then be performed; any fluid that passes into the nose indicates that the obstruction of the nasolacrimal duct has been cleared, and further probing is unnecessary.

If probing does not clear the obstruction, dacryocystorhinostomy is indicated

108
Q

Indication for a Jones tube

A

Insertion of a Jones tube is indicated in patients who have complete obstruction of the lacrimal puncta.

109
Q

Canaliculodacryocystorhinostomy

A

Canaliculodacryocystorhinostomy, with intubation, is recommended for management of strictures found at the junction of the common canaliculus and thelacrimal sac, as well as for obstructions at the level of the canaliculus.

110
Q

Conjunctivorhinostomy

A

Conjunctivorhinostomy is indicated in patients who have absence or obliteration of the tear sac

111
Q

Conjunctivodacryocystostomy

A

Although conjunctivodacryocystostomy is appropriate for patients who have obstruction at the canalicular level, this technique is associated with disruption of the lacrimal sac, resulting in unreliable long-term patency.

112
Q
A 42-year-old woman has drooping of the left eyelid two weeks after undergoing upper eyelid blepharoplasty and injection of botulinum toxin into the forehead. Physical examination shows 3 mm of ptosis of the left eyelid and 13 mm of levator excursion. When the left eyelid is closed voluntarily, the iris shadow can be visualized through the eyelid. The left tarsal crease is elevated 3 mm when compared with the right tarsal crease.Which of the following types of ptosis is the most likely cause of these findings?
(A) Botulinum toxin-induced
(B) Traumatic aponeurotic
(C) Traumatic mechanical
(D) Traumatic myogenic
(E) Traumatic neurogenic
A

(B) Traumatic aponeurotic

113
Q

Traumatic aponeurotic ptosis

A

Traumatic aponeurotic ptosis is characterized by the physical examination findings of good levator function, elevation of the eyelid crease, and the ability to visualize the shadow of the iris with eyelid closure (positive Nesi sign). In this type of ptosis, the levator aponeurosis is detached from the tarsal plate. Early reattachment of the levator is most likely to produce the best result.

114
Q

A 68-year-old woman desires rejuvenation of the eyelids. Physical examination shows excessive skin of the lower eyelids and 2 mm of scleral show bilaterally. Which of the following is the most appropriate management?
(A) Injection of botulinum toxin (Botox) into the lateral canthal region
(B) Carbon dioxide laser resurfacing of the lower eyelids
(C) Lower eyelid blepharoplasty with excision of skin
(D) Lower eyelid blepharoplasty with lateral canthopexy
(E) Transconjunctival blepharoplasty

A

(D) Lower eyelid blepharoplasty with lateral canthopexy

This 68-year-old woman has substantially diminished elasticity of the skin of the lower eyelids. Because of this, she is at increased risk for development of ectropion postoperatively if blepharoplasty with skin excision is performed alone and the laxity of the lower eyelid skin is not corrected. Therefore, the most appropriate management is lower eyelid blepharoplasty combined with lateral canthopexy, which will decrease the laxity of the lower eyelids and lower the risk for ectropion.

115
Q
Three months after undergoing bilateral lower eyelid blepharoplasty, a 45-year-old woman has scleral show, round, sad-looking eyes, and symptoms of ocular irritation, including photophobia and excessive tearing. The most likely cause of these findings is scarring between the orbital septum and which of the following structures?
(A) Capsulopalpebral fascia 
(B) Inferior oblique muscle
(C) Orbicularis oculi muscle 
(D) Orbital rim
(E) Tarsal plate
A

(A) Capsulopalpebral fascia

116
Q

Lower eyelid retraction

A

Lower eyelid retraction is defined as inferior malpositioning of the lower eyelid margin without eyelid eversion. Affected patients have round, sad-looking eyes, scleral show, and symptoms of ocular irritation, including photophobia, excessive tearing, and nocturnal lagophthalmos. Ocular lubricants typically provide only minimal relief.

117
Q

Causes of lower eyelid retraction

A

Potential causes of lower eyelid retraction include scarring between the orbital septum and capsulopalpebral fascia, laxity of the lateral canthal tendon, and descent of the structures of the midface.

118
Q

A 30-year-old woman is undergoing examination six months after sustaining periorbital lacerations in a motor vehicle collision. She has 3.5 mm of ptosis, and levator function is greater than 10 mm. Which of the following is the most appropriate management?
(A) Advancementof MŸeller’s muscle
(B) Eyebrow suspension
(C) Fasanella-Servat procedure
(D) Repositioning of the levator aponeurosis
(E) Levator resection

A

(D) Repositioning of the levator aponeurosis = levator plication

This patient who has ptosis of 3.5 mm with good levator function (greater than 10 mm) is best managed with repositioning of the levator aponeurosis

119
Q

Injuries to the levator complex may be concealed by acute __________,

A

Because injuries to the levator complex may be concealed by acute edema, the surgeon should allow for resolution of the edema, myoneural recovery, and scar softening. After this has occurred, the degree of ptosis and the extent of levator function can be measured in order to determine the appropriate surgical procedure.

120
Q

Eyebrow suspension for ptosis vs levator function

A

Eyebrow suspension is most appropriate for patients who have poor levator function (less than 4 mm) and greater than 3 mm of ptosis.

121
Q

The Fasanella-Servat procedure shortens _________ and is best for:

A

The Fasanella-Servat procedure shortens the lower components of the eyelid (ie, the tarsus, conjunctiva, and MŸller’s muscle). The modification of this procedure spares the muscle and is best for patients who have levator function of greater than 10 mm and ptosis of less than 2 mm

122
Q

Levator resection for ptosis vs levator function

A

Levator resection is reserved for those patients who have levator function between 4 mm and 10 mm, and ptosis of greater than 3 mm.

123
Q

Treatment of upper eyelid ptosis by amount of ptosis and levator fxn

A

[Levator function < 4 mm] + [Ptosis >3 mm] = eyebrow suspension

[Levator function 4-10 mm] + [Ptosis 3 mm] = levator resection / levator advancement

[Levator function > 10 mm] + [Ptosis ~3.5 mm] = levator plication / aponeurotic plication

[Levator function > 10 mm] + [Ptosis 1-2 mm] = Fasanella-Servat

124
Q

Blepharoplasty vs ptosis correction

A

Blepharoplasty is appropriate in patients with excess skin andfat around the eyelids, but not in patients with true ptosis.

125
Q

A 25-year-old man has ectropion of the left lower eyelid and excessive tearing of the eye one month after undergoing reduction of an orbital floor fracture through a subciliary approach. The conjunctiva does not appear to be markedly irritated. Which of the following is the most appropriate management?
(A) Observation with massage and taping of the eyelid
(B) Injection of a corticosteroid
(C) Lateral canthopexy
(D) Scar release and grafting of the conjunctiva using a mucosa graft
(E) Skin-muscle blepharoplasty of the lower eyelid

A

(A) Observation with massage and taping of the eyelid

In this patient who has ectropion of the left lower eyelid and excessive tearing of the left eye one month after undergoing fracture repair, the most appropriate management is observation with massage and taping of the eyelid. Scleral show and ectropion of the lower eyelid occur frequently in patients who have undergone surgery involving the lower eyelid. These complications typically improve over time. At four weeks, collagen is not compact; it will remodel significantly over the following two to six weeks. Operative procedures, such as lateral canthopexy or lamellar release and grafting, are indicated only if there is permanent scarring

126
Q
What is the approximate percentage of patients undergoing reconstruction of fractures of the orbital floor through a transconjunctival approach who will develop ectropion?
(A) 0%
(B) 10%
(C) 20%
(D) 30%
(E) 40%
A

(A) 0%

127
Q

Orbital blowout fractures and ZMC fractures, repaired via preseptal transconjunctival approach: rate of ectropion

A

0%

128
Q

Orbital blowout fractures and ZMC fractures, repaired via subciliary approach: rate of ectropion

A

25%

129
Q

A 62-year-old woman has visual obstruction of the right eye. On examination, she has ptosis of 3 to 4 mm of the right upper eyelid and an elevated supratarsal crease. These findings are most consistent with which of the following conditions?
(A) Dehiscence of the levator aponeurosis
(B) Facial nerve injury
(C) Horner’s syndrome
(D) Myasthenia gravis
(E) Periorbital fat atrophy

A

(A) Dehiscence of the levator aponeurosis

130
Q

The photograph shown above is of a 56-year-old man who underwent open reduction and internal fixation of a malar complex fracture on the right and cranial bone grafting of the right orbital floor three months ago after sustaining bony injuries in a motor vehicle collision. He had no skin lacerations at the time of injury. Which of the following is the most likely cause of the lower eyelid deformity?
(A) Entrapment of Lockwood’s ligament
(B) Inferior displacement of the orbital floor
(C) Loss of skin elasticity
(D) Periorbital fat atrophy
(E) Shortening of the posterior lamella

A

(E) Shortening of the posterior lamella

Entrapment of Lockwood’s ligament would lead to a loss of globe support, and inferior displacement of the cranial bone grafts and orbital floor would result in dystopia. Scleral show and ectropion resulting from excess skin excision are more typical of cosmetic blepharoplasty than internal fixation. Periorbital fat atrophy can result in scleral show and a change in globe position but rarely causes ectropion in patients with traumatic orbital injuries.

131
Q

Composition of the lower lid lamellae

A

The lower eyelid is formed by the anterior, middle, and posterior lamellae:
Anterior lamella: skin and orbicularis oculi muscle.
Middle lamella: orbital septum
Posterior lamella, or capsulopalpebral fascia: the tarsus muscle, lower lid retractors, and conjunctiva.

132
Q

Injury to what in the lower lid most often causes malposition of the lower eyelid?

A

Shortening and scarring of the posterior lamella and septum are most common.

133
Q
A 45-year-old woman has had severe epiphora on the right side for the past four months. Shesustained a comminuted naso-orbital ethmoid fracture when she was struck in the face by a softball six months ago; open reduction and internal fixation were performed immediately after injury. Dacryocystography shows obstruction of the nasolacrimal duct.Which of the following is the most appropriate operative management?
(A) Conjunctivodacryocystostomy
(B) Conjunctivodacryocystorhinostomy
(C) Conjunctivorhinostomy
(D) Dacryocystorhinostomy
(E) Dacryocystostomy
A

(D) Dacryocystorhinostomy

134
Q

Procedure for correction of nasolacrimal duct obstruction and success rate

A

Dacryocystorhinostomy is used for correction of nasolacrimal duct obstruction. Many methods of dacryocystorhinostomy have been described. The single lacrimal flap technique, as well as other techniques that do not involve flaps, has produced long-term patency rates of 90%

135
Q

Procedures for correction of obstruction of the nasolacrimal duct at the canalicular level

A

Conjunctivodacryocystostomy and conjunctivodacryocystorhinostomy are procedures used for reconstruction in a patient who has obstruction at the canalicular level.

136
Q
The patient shown in the photograph above will be at increased risk for development of which of the following complications following four-eyelid blepharoplasty? (The patient shown in the photograph has minimal exophthalmos and moderate scleral show. )
(A) Diplopia
(B) Dry eye syndrome
(C) Entropion
(D) Hematoma
(E) Ptosis
A

(B) Dry eye syndrome

The patient shown in the photograph has minimal exophthalmos and moderate scleral show.
Such clinical findings, as well as proptosis, hypotonia of the lower eyelids, and maxillary hypoplasia, are significant predictors of dry eye syndrome, while low tear film is less predictive of dry eye syndrome.

137
Q

Low tear film vs other symptoms to predict dry eye syndrome

A

not as predictive

138
Q

Clinical findings predictive of dry eye syndrome

A

Exopthalmos, moderate scleral show, proptosis, hypotonia of the lower lids, maxillary hypoplasia

139
Q
The photographs shown above are of a 58-year-old man who has recurrent painless edema of the eyelids. Three upper eyelid blepharoplasty procedures over the past 30 years have not resolved this condition. On physical examination, the skin of the upper eyelids is thin, and results of snap testing are poor.These findings are most consistent with which of the following?
(A) Blepharochalasis
(B) Dermatochalasis
(C) Dry eye syndrome
(D) Pachydermoperiostosis
(E) Senile ptosis
A

(A) Blepharochalasis

140
Q

Dermatochalasis

A

Patients with dermatochalasis have occasional episodes of visual obstruction resulting from excess eyelid skin.

141
Q

Dry eye syndrome

A

Dry eye syndrome is caused by corneal exposure following blepharoplasty and manifests as pain, dryness, and blurred vision.

142
Q

Pachydermoperiostosis

A

Pachydermoperiostosis, or idiopathic hypertrophic osteoarthropathy, is a familial condition of unknown cause characterized by progressive enlargement of the eyelids, hands, feet, and toes. The conjunctivae are covered by hypertrophic papillae. Ptosis and visual obstruction are common.

143
Q
A 45-year-old woman is being evaluated after undergoing upper eyelid blepharoplasty. Examination shows persistent fullness involving the lateral third of the upper eyelids. These findings are most consistent with which of the following?
(A) Descending lacrimal glands
(B) Eyebrow ptosis
(C) Lateral compartment fat
(D) Subcutaneous fat deposits
(E) Supraorbital bony prominences
A

(A) Descending lacrimal glands

In this patient who has undergone upper eyelid blepharoplasty, the persistent upper eyelid fullness is most likely a result of descending lacrimal glands. Appropriate management of this condition can include resuspension of the glands, which has been advocated by some surgeons

144
Q

Why is excision of the lacrimal glands not recommended

A

Excision of the glands is not recommended because it may result in the development of keratoconjunctivitis sicca

145
Q

Difference in clinical presentation of descending lacrimal glands vs eyebrow ptosis

A

Eyebrow ptosis would most likely be seen over the entire eyebrow, not just the lateral third.

146
Q
A 43-year-old woman has miosis, anhidrosis,and blepharoptosis measuring 2 mm. On examination, the eyelid crease is normal and function of the levator muscle is good. Which of the following is the most likely diagnosis?
(A) Blepharophimosis syndrome
(B) Congenital ptosis
(C) Horner's syndrome
(D) Involutional ptosis
(E) Myasthenia gravis
A

(C) Horner’s syndrome

147
Q

What causes Horner’s syndrome?

A

sympathetic denervation of the superior cervical ganglion

148
Q

Blepharophimosis syndrome

A

Blepharophimosis syndrome is a congenital condition consisting of ptosis, telecanthus, and phimosis of the upper eyelid fissure

149
Q
A 40-year-old woman has steady, lancinating pain in the globe and orbit and episodes of vomiting six hours after undergoing blepharoplasty of the lower eyelids. She says that she sees sparkles and flashes and has the sensation similar to a "window shade" closing over the lower half of her range of vision. These findings are most consistent with which of the following?
(A) Acute glaucoma
(B) Adverse effects of anesthesia
(C) Migraine
(D) Retrobulbar hematoma
(E) Transient ischemic attack
A

(D) Retrobulbar hematoma

150
Q

Retrobulbar hematoma symptoms

A

Retrobulbar hematoma is mostfrequently characterized by steady, severe, lancinating pain in the globe and orbit (mimicking symptoms of acute glaucoma), which can occur alone or with scintillating scotomas (ie, sparkles and flashes, mimicking the symptoms of severe migraine) and hemianopsia or amaurosis fugax (ie, findings similar to a “window shade” pulled over the lower half of the visual field, mimicking a transient ischemic attack). Other symptoms associated with the development of hematoma following blepharoplasty include early discharge from the eye, perioperative and postoperative vomiting, and coughing. The use of aspirin-containing products has also been associated

151
Q

Appropriate management of retrobulbar hematoma

A

Appropriate management includes surgical exploration and lateral canthotomy, with ophthalmologic consultation. Mannitol and carbonic anhydrase inhibitors can also be administered to decrease intraocular pressure and reestablish blood flow

152
Q
After undergoing repair of an orbital fracture, a patient has progressive loss of vision resulting from the development of a retrobulbar hematoma. In order to immediately relieve intraocular pressure, which of the following structures should be released?
(A) Lateral canthal tendon
(B) Levator aponeurosis
(C) Orbicularis oculi muscle
(D) Tarsal plate
(E) Tenon's capsule
A

(A) Lateral canthal tendon

153
Q

Why do some structures not need to be released for retrobulbar hematoma?

A

Eyelid structures such as the levator aponeurosis, orbicularis oculi muscle, and tarsal plate do not need to be released. Because these structures lie external to the support sling created by the lateral canthal tendon and Lockwood’s suspensory ligament, they already have at least partial mobility

154
Q

Why detach the lateral cantonal tendon versus Lockwood’s suspensory ligament for retrobulbar hematoma?

A

Tenon’s capsule covers the globe and extraocular muscles, creating a barrier between these structures and the orbital fat. The lower portion of the capsule comprises Lockwood’s suspensory ligament. Because of their position proximal to the globe, it is more prudent to detach the lateral canthal tendon from the bony rim in order to allow for advancement of the intraorbital contents. Surgical detachment of the lateral canthal tendon will disrupt the globe only minimally. Any loss of lower eyelid height can be restored later with lateral canthopexy.

155
Q
Which of the following bones comprises the greatest portion of the medial orbital wall?
(A) Ethmoid
(B) Lacrimal
(C) Maxilla
(D) Palatine
(E) Sphenoid
A

(A) Ethmoid

156
Q

The medial orbital wall is comprised primarily of:

A

The medial orbital wall is comprised primarily of the orbital plate of the ethmoid bone.

157
Q

Composition of the ethmoid bone

A

This bone is made up of a horizontal or cribriform plate, a midline perpendicular plate that forms the nasal septum, and symmetric lateral masses. The outer wall of each lateral mass is the medial orbital wall, the inner walls are the sidewalls of the nasal

158
Q

Fractures related to orbital blowouts tend to involve:

A

Knowledge of the anatomy of the medial orbital wall is important when diagnosing and treating extensive orbital blowout fractures, which often include the orbital floor. These fractures can extend to involve the inferomedial hillock and central section of the medial orbital wall; if this occurs, the orbital contents may be displaced into the maxillary and ethmoid sinuses

159
Q

When is it most likely that a patient will develop post traumatic enopthalmos after orbital blowout fracture?

A

If the anatomic volume of the bony orbit is not restored surgically, the patient may develop posttraumatic enophthalmos and diplopia. This will most likely occur in those patients who have an increase in bony orbital volume of greater than 5%

160
Q

The orbital floor is comprised of:

A

The orbital floor is comprised of the maxilla medially and the zygoma anteriorly.

161
Q
A 48-year-old man has traumatic telecanthus after sustaining a naso-orbito-ethmoid fracture. Open reduction and internal fixation are performed with transnasal wiring of the fracture fragments surrounding the medial canthal tendon. At follow-up examination one year after the initial injury, the patient has epiphora of the right eye. Findings on Jones I dye testing are negative and Jones II dye testing are positive. Which of the following anatomic sites is the most likely origin of this patient's findings?
(A) Canaliculi
(B) Nasolacrimal duct
(C) Lacrimal gland
(D) Punctum
(E) Superior meatus
A

(B) Nasolacrimal duct

162
Q

The posterior ethmoid air cells empty into:

A

the superior meatus

163
Q

The nasolacrimal duct empties into:

A

the inferior meatus

164
Q
A 2-year-old child is being evaluated because he has deformities of the eyelids and upper face. Examination shows large epicanthal folds of the lower eyelids with epicanthus inversus, horizontal shortening of the eyelids, and 5 mm of ptosis bilaterally. Levator excursion is 4 mm. These findings are most consistent with
(A) blepharochalasis
(B) blepharophimosis syndrome
(C) blepharospasm
(D) congenital epicanthus
(E) epiblepharon
A

(B) blepharophimosis syndrome

165
Q

Blepharophimosis syndrome

A

Blepharophimosis syndrome, which is classified according to three types, is a form of congenital ptosis.

Blepharophimosis syndrome, type 1: large epicanthal folds with epicanthus inversus, horizontally shortened eyelids, and severe ptosis.

Blepharophimosis syndrome, type 2: telecanthus, absence of the epicanthal folds, severe bilateral ptosis, absence of levator function, and skin shortage involving all four lids.

Blepharophimosis syndrome, type 3: absence of epicanthal folds, telecanthus, an antimongoloid slant of the palpebral fissures, severe ptosis, mild orbital hypertelorism, and skin deficiencies.

166
Q

Severe ptosis is associated with blepharophimosis syndrome type ___

A

The presence of severe congenital ptosis leads to a diagnosis of blepharophimosis syndrome type 1

167
Q

Surgical correction of blepharophimosis syndrome involves :

A

Surgical correction of blepharophimosis syndrome involves repair of the epicanthal folds and correction of eyelid ptosis. Levator resection, medial canthoplasty, and fascial suspension techniques have all been used in combination to correct the abnormalities. A five-flap technique that combines double Z-plasties and a Y-to-V flap is most often used for repair of the epicanthal folds. Ideally, the canthus should lie halfway between the pupil and the center of the nasal bridge following all repair procedures

168
Q

Blepharospasm

A

Blepharospasm is a condition in which there is sustained, forced, involuntary closing of the eyelids.

169
Q
A 68-year-old woman seeks correction of drooping eyelids and impaired upward gaze. Physical examination shows excessive hooding of the upper eyelid skin; visual field testing confirms obstruction in the upper fields. Levator excursion is 14 mm bilaterally. There is 2 mm of ptosis of the left eyelid; the right eye is unaffected.In addition to blepharoplasty, which of the following is the most appropriate management?
(A) Division of MŸller's muscle
(B) Fasanella-Servat procedure
(C) Fascial sling
(D) Levator advancement
(E) Resection of the levator muscle
A

(D) Levator advancement

Bilateral blepharoplasty with fat pad removal and ptosis repair using levator advancement will address this woman’s visual field defect and mild ptosis. These procedures are used for patients with normal levator function (defined as greater than 10 mm). Bilateral upper eyelid blepharoplasty alone or in conjunction with fat pad removal would not correct the ptosis, while repair of the ptosis only would not address the visual field obstruction.

The Fasanella-Servat procedure is used to correct minimal ptosis but is a more difficult, complicated procedure than levator plication.

170
Q

A 50-year-old woman has right eyelid ptosis of 2 mm two days after undergoing uncomplicated four-eyelid blepharoplasty under local anesthesia. On physical examination, there is moderate edema of the upper and lower eyelids. Which of the following is the most appropriate next step in management?
(A) Reassurance and continued follow-up examinations
(B) Eyelid massage and stretching exercises
(C) Administration of phenylephrine eyedrops
D) Immediate operative exploration of the eyelid
(E) Levator plication seven days after the initial procedure

A

(A) Reassurance and continued follow-up examinations

Although operative exploration is warranted in patients who have eyelid discrepancies following blepharoplasty, it would not be useful in this patient who did not initially undergo repair of the levator mechanism. Instead, surgical treatment in this patient should be delayed for two to six months to allow for spontaneous recovery.

171
Q

Ptosis of a mild to moderate degree after blepharoplasty

A

Ptosis of a mild to moderate degree is a common finding following blepharoplasty; common causes include postoperative edema of the eyelids and hemorrhage into MŸller’s muscle. Because these complications generally resolve spontaneously over time, reassurance and observation with frequent follow-up examinations are most appropriate.

172
Q

A 25-year-old man has ectropion and excessive scleral show one year after sustaining a chemical burn of the lower right eyelid, which was allowed to heal without surgical intervention. He currently uses ocular ointments daily. Which of the following is the most appropriate management?
(A) Scar massage and intralesional injection of a corticosteroid
(B) Full-thickness skin grafting
(C) Insertion of a gold eyelid weight
(D) Lateral canthoplasty
(E) Lateral tarsal wedge excision

A

(B) Full-thickness skin grafting

173
Q

A 70-year-old woman has a skin defect with a diameter of 1 cm after undergoing resection of a basal cell carcinoma of the right upper eyelid. Primary closure of the defect is not possible. Which of the following is most appropriate for cutaneous full-thickness coverage of the defect?
(A) Cross-lid flap
(B) Retroauricular skin graft
(C) Skin graft from the contralateral upper eyelid
(D) Supraclavicular skin graft
(E) Wedge resection and primary closure of the remaining eyelid

A

(C) Skin graft from the contralateral upper eyelid

A skin graft from the contralateral upper eyelid should be used for full-thickness coverage of this patient’s defect. Because older patients typically have dermatochalasia, there is often sufficient skin in the unaffected upper eyelid that can be harvested and used to cover small skin deficits.

A cross-lid flap is more appropriate forcoverage of a full-thickness defect.

174
Q
A 65-year-old woman comes to the office for consultation on blepharoplasty and rhytidectomy. She notes that her eyelid creases are asymmetric, with the right upper crease higher than the left. She also has difficulty seeing out of her right eye when she is tired. She notes that these symptoms have worsened progressively over the past 5 years. She has levator function of 11 mm with the brow and frontalis muscle in neutral position. This patient most likely has which of the following types of ptosis?
A) Involutional
B) Mechanical
C) Myogenic
D) Neurogenic
E) Traumatic
A

A) Involutional

The most likely diagnosis is involutional or senile ptosis. In evaluating a patient requesting a blepharoplasty, one must also evaluate for blepharoptosis. If a blepharoplasty is performed without correction of ptosis, the patient will have continuing ptosis and potentially be dissatisfied.

The most common type of blepharoptosis is involutional or senile ptosis. A thorough history and physical examination should assess ophthalmologic and neurologic causes. On physical examination, there is a characteristic high skin crease (greater than 7 mm), thinned upper eyelid, and lid drop on downward gaze. The levator function (amount of lid excursion with the brow and frontalis muscle held in neutral position) should be assessed. Normal function is greater than 10 mm, moderate function is 5 to 10 mm, and poor function is less than 5 mm.

The etiology of ptosis can be classified into neurogenic (oculomotor nerve palsy, Horner syndrome, Marcus Gunn jaw-winking syndrome), myogenic (myasthenia gravis, myotonic dystrophy, mitochondriopathy), mechanical (edema or tumors), traumatic (birth trauma, muscle or nerve damage), congenital, or neurotoxic (such as in envenomation, snake bites, or botulism). None of these are likely, considering this patient’s history and physical examination.

175
Q

A 60-year-old man undergoes Mohs micrographic resection of an upper eyelid squamous cell carcinoma. A photograph is shown. Which of the following reconstruction procedures will provide the most functional and aesthetically pleasing result in this patient?
(100% of upper lid gone)

A) Free nasoseptal grafting
B) Reconstruction with a forehead flap
C) Reconstruction with a free flap
D) Reconstruction with a glabellar flap
E) Reconstruction with a lid switch flap
A

E) Reconstruction with a lid switch flap

A Cutler-Beard flap is a two-stage lid switch flap, taken from the lower lid.

Some of its disadvantages include two-stage reconstruction requiring eye occlusion for a number of weeks, sacrifice of lower lid tissue (and, commonly, a subsequent ectopion), and lack of intrinsic support. Many modifications have improved on its design including those that provide support. There are many reports of other types of flaps, but for a defect of this size and the anatomic requirements, the best choice is a lid switch flap.

176
Q

A 50-year-old woman comes to the office 2 weeks after receiving botulinum toxin type A injections for forehead rhytids. She is pleased with the results but has developed unilateral eyelid ptosis. Apraclonidine 0.5% eyedrops are prescribed to improve upper eyelid function. Which of the following best describes the mechanism for improvement of upper eyelid ptosis in this patient?
A) Alpha-2 adrenergic stimulation of the Müller muscle of the upper eyelid
B) Beta-2 adrenergic stimulation of the levator palpebrae superioris
C) Muscarinic parasympathetic inhibition of the levator palpebrae superioris
D) Nicotinic parasympathetic inhibition of the Müller muscle of the upper eyelid

A

A) Alpha-2 adrenergic stimulation of the Müller muscle of the upper eyelid

Apraclonidine is an alpha-2 receptor agonist and is believed to increase muscle tone of the sympathetically innervated Müller muscle located in the upper eyelid.

177
Q

Mechanism / use of apraclonidine

A

Apraclonidine is an alpha-2 receptor agonist and is believed to increase muscle tone of the sympathetically innervated Müller muscle located in the upper eyelid, for botox-induced ptosis

178
Q
A 49-year-old woman is evaluated because of a traumatic laceration of the right lower eyelid and cheek. Physical examination shows difficulty with eyelid closure, voluntary squinting, and animation. Which of the following branches of the facial nerve is most likely injured?
A) Buccal
B) Cervical
C) Marginal mandibular
D) Temporal
E) Zygomatic
A

E) Zygomatic

Anatomically, the orbicularis oculi muscle is divided into three segments: pretarsal, preseptal, and orbital. However, functionally, the orbicularis oculi muscle is divided into the medial inner canthal orbicularis and the extracanthal orbicularis. The medial inner canthal orbicularis is responsible for blinking, lower lid tone, and the pumping mechanism of the lacrimal system. Innervation to the inner canthal orbicularis is from the buccal branches of the facial nerve. The zygomatic branch of the facial nerve innervates the extracanthal orbicularis, which controls eyelid closure, voluntary squinting, and animation. The temporal, marginal mandibular, and cervical branches do not provide innervation to the orbicularis oculi muscle.

179
Q

A 65-year-old woman has inadequate tear secretion because of a poorly functioning lacrimal gland. Which of the following aspects of this patient’s tear film is most likely to be affected?
A) The amount of lipid in the tear film
B) The amount of mucin in the tear film
C) The antimicrobial property of the tear film
D) The degree of evaporation of the tear film
E) The dispersion of the tear film

A

C) The antimicrobial property of the tear film

Tears are a trilaminar fluid. The precorneal layer is formed by mucin-secreting goblet cells in the conjunctiva. This inner layer of the tear film covers the cornea and promotes the dispersion of the overlying aqueous layer. The meibomian glands produce the outer lipid layer. This oil layer helps to prevent the evaporation of the tear film.

The lacrimal gland secretes the middle layer. This aqueous layer is made of water and proteins. This layer promotes osmotic regulation and the control of infectious agents. As a result, dysfunction can result in dryness and an increase in infections.

180
Q

Eye secretions: what does the lacrimal gland secrete?

A

The lacrimal gland secretes the middle layer. This aqueous layer is made of water and proteins. This layer promotes osmotic regulation and the control of infectious agents. As a result, dysfunction can result in dryness and an increase in infections.

181
Q

The mucin-secreting goblet cells of the conjunctiva secrete:

A

The precorneal layer is formed by mucin-secreting goblet cells in the conjunctiva. This inner layer of the tear film covers the cornea and promotes the dispersion of the overlying aqueous layer.

182
Q

The meibomian glands produce:

A

The meibomian glands produce the outer lipid layer. This oil layer helps to prevent the evaporation of the tear film.

183
Q
A 65-year-old woman desires correction of her “sleepy eyes.” Physical examination shows bilateral moderate involutional ptosis of the upper eyelids. Mild lateral displacement of the tarsal plate is noted. Repair of the levator aponeurosis using an anterior approach is planned. Which of the following best describes the proper vertical plane to position the lifting suture in a single-suture technique?
A) Apex of the tarsal plate
B) Lateral limbus
C) Medial limbus
D) Mid pupil
E) Midline of central fat compartment
A

D) Mid pupil

To achieve proper contour, the primary lifting suture should be placed on the vertical plane of the mid pupil. It is not uncommon for a patient with involutional ptosis to also have lateral displacement of the tarsal plate. If the central lifting suture is placed at the apex of the tarsal plate, the contour will be abnormally shifted laterally. The medial and lateral limbus position will also cause abnormal contour shifts. The midline of the central fat compartment has an inconsistent location in relation to the pupil.

184
Q
A 4-year-old boy is brought to the office for treatment and evaluation of lid ptosis. On examination, bilateral lagophthalmos, poor levator excursion, and severe ptosis are noted. Which of the following is the most likely diagnosis?
A) Blepharophimosis syndrome
B) Congenital epiblepharon
C) Congenital euryblepharon
D) Fraser cryptophthalmos syndrome
E) Treacher Collins syndrome
A

A) Blepharophimosis syndrome

Blepharophimosis syndrome is the only diagnosis listed that is associated with congenital ptosis.

Blepharophimosis syndrome is associated with a tetrad of findings including ptosis, telecanthus, epicanthus inversus, and decreased horizontal lid fissure. In type I blepharophimosis, patients have epicanthus inversus and ptosis. In type II, findings include telecanthus, ptosis, ectropion of the lower lids, absent epicanthal folds, and insufficient skin in all lids. Type III is notable for telecanthus, ptosis, hypertelorism, slanting palpebral fissures, and insufficient eyelid skin. Correction involves a variety of techniques including, but not limited to, Z-plasty, transnasal wiring of the medial canthal tendon, and ptosis correction with frontalis suspension. Other abnormalities of the blepharophimosis syndrome include flattening of the nasal dorsum, hypoplasia of the superior orbital rim, as well as forehead and ear deformities.

In epiblepharon, the eyelashes are vertical as a result of excess pretarsal muscle and skin overriding the margin of the eyelid, often affecting the lower lids. This causes corneal irritation. If the condition does not resolve spontaneously in the first few years of life, correction involves shortening of the anterior lamella through excision of a horizontal piece of skin and orbicular muscle. Epiblepharon may also be caused by trauma, burns, or fractures.

Euryblepharon refers to widening of the palpebral fissure both laterally and vertically caused by a shortage of eyelid tissue. Treatment involves corneal protection and may require surgical correction with standard techniques used for ectropion repair.

Cryptophthalmos is a failure in embryonic development of the lid fold. The eye is buried in the developing cover of the epithelium and does not differentiate normally. It may be associated with other congenital abnormalities such as syndactyly, cardiac, facial, and ear defects.

Treacher Collins syndrome is a maxillary-zygomatic cleft with a coloboma of the lower eyelid and absent eyelashes.

185
Q

Blepharophimosis syndrome

A

Blepharophimosis syndrome is associated with a tetrad of findings including ptosis, telecanthus, epicanthus inversus, and decreased horizontal lid fissure.

186
Q

Epiblepharon

A

In epiblepharon, the eyelashes are vertical as a result of excess pretarsal muscle and skin overriding the margin of the eyelid, often affecting the lower lids. This causes corneal irritation. If the condition does not resolve spontaneously in the first few years of life, correction involves shortening of the anterior lamella through excision of a horizontal piece of skin and orbicular muscle. Epiblepharon may also be caused by trauma, burns, or fractures.

187
Q

Euryblepharon

A

Euryblepharon refers to widening of the palpebral fissure both laterally and vertically caused by a shortage of eyelid tissue. Treatment involves corneal protection and may require surgical correction with standard techniques used for ectropion repair.

188
Q

Cryptopthalmos

A

Cryptophthalmos is a failure in embryonic development of the lid fold. The eye is buried in the developing cover of the epithelium and does not differentiate normally. It may be associated with other congenital abnormalities such as syndactyly, cardiac, facial, and ear defects.

189
Q
A 26-month-old boy is brought to the office for evaluation because of worsening congenital ptosis. A photograph is shown. Which of the following is the most appropriate next step in management?
(ptotic right eye)
A) Eye lubrication
B) MRI
C) Patching of the non-affected eye
D) Surgical correction
E) Observation only
A

B) MRI

In most cases of congenital ptosis, a droopy eyelid results from a localized myogenic dysgenesis. Rather than normal muscle fibers, fibrous and adipose tissues are present in the muscle belly, diminishing the ability of the levator to contract and relax. Therefore, the condition is commonly called congenital myogenic ptosis. Most cases of congenital ptosis are idiopathic.

Surgical correction of congenital ptosis can be undertaken at any age depending on the severity of the disease. Earlier intervention may be required if significant amblyopia or ocular torticollis is present. If intervention is not urgent, surgery is often delayed until age 3 to 4 years. Waiting until this age allows for more accurate measurements preoperatively.

Congenital ptosis can also occur when the innervation to the levator is interrupted through neurologic or neuromuscular junction dysfunction. Nerve compression by external forces such as tumor must be ruled out. Specifically, when ptosis presents acutely or subacutely in a child over 1 year of age, compression of cranial nerve III is a concern.

Rhabdomyosarcoma is the most common primary malignancy of the orbit in children. MRI shows a well-circumscribed mass that typically enhances with gadolinium. On T1-weighted imaging, the tumor usually appears isointense to extraocular muscles but hypointense to orbital fat. On T2-weighted imaging, the lesion appears hyperintense to extraocular muscles and orbital fat. This tumor can grow rapidly and is treated with combined chemotherapy and radiation.

190
Q

Most common cause of congenital ptosis

A

In most cases of congenital ptosis, a droopy eyelid results from a localized myogenic dysgenesis. Rather than normal muscle fibers, fibrous and adipose tissues are present in the muscle belly, diminishing the ability of the levator to contract and relax. Therefore, the condition is commonly called congenital myogenic ptosis. Most cases of congenital ptosis are idiopathic.

191
Q

A 48-year-old woman comes to the clinic with new left eyelid ptosis 1 week after receiving injections of botulinum toxin type A for treatment of glabellar rhytides. The patient is prescribed apraclonidine ophthalmic solution for the left eye. Complete resolution is noted 2 days later. Which of the following is the most important mechanism by which apraclonidine resulted in improvement of this patient’s eyelid ptosis?
A) Displacement of botulinum toxin type A from its presynaptic receptors
B) Inhibition of alpha-adrenergic receptors in the levator palpebrae superioris muscle
C) Inhibition of alpha-adrenergic receptors in the superior tarsal (Müller) muscle
D) Stimulation of alpha-adrenergic receptors in the levator palpebrae superioris muscle
E) Stimulation of alpha-adrenergic receptors in the superior tarsal (Müller) muscle

A

E) Stimulation of alpha-adrenergic receptors in the superior tarsal (Müller) muscle

The most important mechanism by which apraclonidine caused improvement of this patient’s eyelid ptosis as described is stimulation of alpha-adrenergic receptors in the superior tarsal (Müller) muscle.

Botulinum toxin type A is a protease that, through degradation of the SNAP-25 protein within axonal terminals, prevents fusion of cytoplasmic vesicles to the presynaptic membrane and subsequent release of neurotransmitters, in particular acetylcholine (ACh). Release of ACh into the synaptic cleft is necessary for normal skeletal muscle contraction.

Transient eyelid ptosis is a potential side effect of injection of botulinum toxin type A into the upper third of the face, with reported incidence between 2 and 11%. It occurs when the injected toxin migrates through the orbital septum and reaches the levator palpebrae superioris (LPS) muscle, a skeletal muscle innervated by the oculomotor (III) nerve.

The superior tarsal (Müller) muscle acts in conjunction with LPS to elevate the upper eyelid. Müller muscle is a smooth muscle, innervated by the sympathetic nervous system via alpha-adrenergic receptors. Stimulation of these receptors (as caused by apraclonidine – a selective alpha-2 receptor agonist with weak alpha-1 activity) can compensate for partial LPS dysfunction, correcting 1 to 3 mm of eyelid ptosis.

Apraclonidine is not known to cause significant displacement of botulinum toxin type A from its presynaptic receptors. Contraction of the LPS muscle, as for other skeletal muscles, occurs by stimulation of its cholinergic receptors within the neuromuscular junctions. Inhibition of alpha-adrenergic receptors in Müller muscle would cause it to relax, increasing the upper eyelid ptosis.

192
Q

Mechanism of botulinum toxin type A

A

Botulinum toxin type A is a protease that, through degradation of the SNAP-25 protein within axonal terminals, prevents fusion of cytoplasmic vesicles to the presynaptic membrane and subsequent release of neurotransmitters, in particular acetylcholine (ACh). Release of ACh into the synaptic cleft is necessary for normal skeletal muscle contraction.

193
Q
A 45-year-old man comes to the office because of frequent blinking and squeezing of the eyelids. Examination shows idiopathic blepharospasm. Which of the following is the most appropriate management?
A) Administration of a benzodiazepine
B) Injection of botulinum toxin type A
C) Myectomy
D) Neurectomy
E) Use of tinted glasses
A

B) Injection of botulinum toxin type A

Blepharospasm is an involuntary eye movement disorder characterized by frequent blinking and squeezing of the eyelids. Blepharospasm is treated with botulinum toxin type A. Treatment usually brings some relief, although patients require constant monitoring and re-treatment with botulinum toxin type A. Use of tinted glasses may reduce a trigger of essential blepharospasm but is not the primary treatment. Benzodiazepines are not used as the first-line management of essential blepharospasm. Myectomy and neurectomy are reserved for severe cases that are refractory to medical management and are therefore not indicated unless the patient fails all other forms of management.

194
Q
A 56-year-old woman undergoes resection of the conjunctiva and Müller muscle for treatment of blepharoptosis. Which of the following structures is encountered immediately anterior to the Müller muscle?
A) Capsulopalpebral fascia
B) Central fat pad
C) Levator palpebrae superioris muscle
D) Lockwood ligament
E) Retro-orbicularis oculi fat
A

C) Levator palpebrae superioris muscle

The layers of the upper eyelid, in order from superficial to deep, are skin, orbicularis oculi muscle, retro-orbicularis oculi fat, orbital septum, orbital fat (central and nasal or medial in the upper eyelid), levator palpebrae superioris muscle and aponeurosis, Müller muscle, and the conjunctiva. In a Fasanella-Servat procedure for the correction of eyelid ptosis, the conjunctiva and Müller muscle are grasped between clamps just above the border of the tarsal plate. The tissue in the clamp is then excised and closed, thus resecting the Müller muscle and conjunctiva. At the base of the wound after the resection is the levator muscle.

The capsulopalpebral fascia and Lockwood ligament are part of the lower eyelid and orbital contents. The capsulopalpebral fascia inserts on the inferior border of the tarsus. It makes up the anterior superior portion of the lower eyelid retractors distal to the Lockwood ligament. The Lockwood ligament is a fascial thickening that supports the globe. It surrounds the inferior rectus and inferior oblique muscles and fuses with the capsulopalpebral fascia. It is analogous to the Whitnall ligament in the upper eyelid.

195
Q

The layers of the upper eyelid in order from superficial to deep

A

The layers of the upper eyelid, in order from superficial to deep, are skin, orbicularis oculi muscle, retro-orbicularis oculi fat, orbital septum, orbital fat (central and nasal or medial in the upper eyelid), levator palpebrae superioris muscle and aponeurosis, Müller muscle, and the conjunctiva.

196
Q
A 1-year-old male infant is brought to the office because of congenital ptosis of the left eye. On examination, the eyelid margin covers 4 mm of the upper limbus, and levator excursion is 4 mm. Which of the following is the most appropriate treatment?
A ) Fasanella-Servat procedure
B ) Frontalis suspension with sling
C ) Levator advancement
D ) Müller muscle resection
E ) Observation
A

B ) Frontalis suspension with sling

With more than 4 mm of the upper limbus covered by the eyelid margin, the ptosis is considered severe, and with levator function at 4 mm, it is considered poor. Therefore, a frontalis suspension is the appropriate procedure. It can achieve excellent symmetry and long-lasting results. Most patients with congenital ptosis have severe ptosis and poor levator function and will need frontalis suspension.

Ptosis is defined by how much the upper limbus is covered by the lid margin at rest and forward gaze. It is 1 to 2 mm normally.

Ptosis:
Mild = 2 mm
Moderate = 3 mm
Severe = 4+ mm

Levator function:
Excellent = 12–15 mm
Good = 8–12 mm
Fair = 5–7 mm
Poor = 2–4 mm

In terms of treatment:
Fasanella-Servat: mild ptosis and good levator function
Frontalis suspension: severe ptosis and poor levator function
Levator advancement: moderate ptosis and fair levator function
Müller muscle resection: mild ptosis, fair to good levator function

197
Q

How much of the upper limbus usually covered by the lid margin at rest and forward gaze?

A

1-2 mm

198
Q

Grades of ptosis

A

Ptosis:
Mild = 2 mm
Moderate = 3 mm
Severe = 4+ mm

199
Q

Grades of levator function

A
Levator function:
Excellent = 12–15 mm
Good = 8–12 mm
Fair = 5–7 mm
Poor = 2–4 mm
200
Q

When to perform Fasanella-servat

A

Fasanella-Servat: mild ptosis and good levator function

201
Q

When to perform frontal suspension

A

Frontalis suspension: severe ptosis and poor levator function

202
Q

When to perform levator advancement

A

Levator advancement: moderate ptosis and fair levator function

203
Q

When to perform muller muscle resection

A

Müller muscle resection: mild ptosis, fair to good levator function

204
Q

What to perform for mild ptosis and good levator function?

A

Fasanella-Servat

205
Q

What to perform for severe ptosis and poor levator function?

A

Frontalis suspension

206
Q

What to perform for moderate ptosis and fair levator function?

A

Levator advancement

207
Q

What to perform for mild ptosis and fair to good levator function?

A

Muller muscle resection

208
Q

A 55-year-old man comes for evaluation of epiphora in the right eye 6 months after sustaining a soft-tissue injury to the face in a motor vehicle collision. Physical examination shows that the punctum and lacrimal sac are open. A picture taken at the time of injury is shown. Results of Jones I and Jones II testing are negative. No other abnormalities are noted. Which of the following is the most appropriate next step in management?
A ) Botulinum toxin type A injections of lacrimal gland
B ) Conjunctivodacryocystorhinostomy
C ) Dacryocystorhinostomy
D ) Dilation of lacrimal apparatus
E ) Placement of lacrimal stent

A

C ) Dacryocystorhinostomy

Abnormalities of tear drainage may be subdivided into functional and anatomical. Functional failure is related to poor lacrimal pump function, which may be due to a displaced punctum, eyelid laxity, weak orbicular muscle of the eye, or cranial nerve VII palsy. Anatomical obstruction may occur at any point along the lacrimal drainage pathway and may be congenital or acquired. Primary acquired lacrimal duct obstruction occurs in the elderly as a result of fibrosis. Secondary acquired obstruction causes include neoplastic, traumatic, and mechanical mechanisms.

Conjunctivodacryocystorhinostomy is performed in cases of flaccid canaliculi, paralysis of lacrimal pump, and when the site of obstruction is proximal (punctum, canaliculi, lacrimal sac). It is not required when these structures are intact. The dacryocystorhinostomy procedure, which involves fistulization of the lacrimal sac into the nasal cavity, alleviates the symptoms of epiphora.

The lacrimal probe demonstrates that the punctum and the sac are intact. The negative result on both Jones I and II – failure of any fluorescein dye to show up in the nasopharynx – demonstrates complete obstruction distal to the lacrimal sac. Dilation or stent placement is not possible.

With a history of trauma and the laceration shown, this patient does not have epiphora secondary to excess tear production.

209
Q
A 76-year-old man comes to the office for consultation regarding a 5-year history of upper eyelid ptosis. He has no history of serious illness or trauma and takes no medications. Which of the following is the most likely cause?
A ) Levator dehiscence
B ) Muscular dystrophy
C ) Myoneural dysfunction
D ) Oculomotor (III) nerve palsy
E ) Paralysis of the Müller muscle
A

A ) Levator dehiscence

Elderly patients most commonly have symptoms of ptosis from dehiscence of the levator aponeurosis insertion. The resting eyelid position migrates inferiorly as the supratarsal crease moves to a more superior position. Muscular dystrophy affecting the extraocular muscles, chronic progressive external ophthalmoplegia, may affect extraocular muscles and the levator. Nonetheless, this is infrequent relative to levator dehiscence. Myoneural dysfunction that worsens with fatigue or at the end of the day is pathognomonic for myasthenia gravis. This disorder primarily affects young women and may be identified by improvement with neostigmine or edrophonium administration. Paralysis of the Müller muscle may occur following injury to sympathetic inflow, as seen in Horner syndrome.

210
Q
A 67-year-old woman undergoes bilateral reattachment of the levator muscles because of eyelid ptosis 6 months after undergoing cataract surgery. During the procedure, the levator aponeurosis is reattached to which of the following structures?
A ) Lockwood ligament
B ) Müller muscle
C ) Orbital septum
D ) Tarsal plate
E ) Whitnall ligament
A

A ) Lockwood ligament

The levator aponeurosis attaches to the tarsal plate and is detached or attenuated in senile ptosis. The levator aponeurosis is reattached in senile ptosis procedures. The Lockwood ligament is found in the lower eyelid. It forms a “hammock” stretching below the eyeball between the medial and lateral check ligaments and enclosing the inferior rectus and inferior oblique muscles. The Müller muscle is an accessory levator. The orbital septum lies anterior to the fat pads and the levator aponeurosis. The Whitnall ligament is a condensation of the sheath overlying the anterior superior part of the levator muscle. Injury to the levator muscle during cataract surgery and retractors to the open eye can cause traumatic dehiscence of the levator muscle.

211
Q

A 46-year-old woman comes to the office because she cannot completely close her right eye 1 week following bilateral upper eyelid blepharoplasty with levator advancement. Physical examination shows overcorrection of the right eyelid by 2 mm. Slit lamp examination shows no abnormalities. Which of the following is the most appropriate next step in management?
A ) Perform a downward massage of the right eyelid
B ) Perform an edrophonium (Tensilon) test
C ) Perform skin grafting to the right eyelid
D ) Remove the skin and levator sutures
E ) Revise the left eyelid to obtain symmetry

A

A ) Perform a downward massage of the right eyelid

The ability to predict postoperative levels in ptosis surgery has been refined over the years, but there is no completely reliable formula to predict the final tension of the upper eyelid that determines the final upper eyelid level. Revisions are common. Aponeurotic repair is not a measured resection procedure. It does not require patient cooperation and can be performed during general anesthesia.

Early management of postoperative asymmetry includes conservative treatment with downward massage. Aggressive lubrication should be used to protect the cornea. The Tensilon test is useful in the diagnosis of ocular myasthenia. Skin grafting is not appropriate since there is no shortage in skin. Removal of the levator sutures will not correct the problem, and could result in ptosis recurrence. Revision of the left eyelid will lead to dry eye symptoms and unfavorable aesthetics.

212
Q

What is a coloboma?

A

Coloboma is a congenital ocular defect of the eyelid, iris, retina, choroid, or optic disk.

213
Q

Origin of coloboma?

A

The defects can range in size from a small notch to a large structural cleft. Palpebral colobomas are thought to arise from a localized growth disturbance, while colobomas of the iris, retina, and optic disk arise from defective closure of the optic fissure.

214
Q
A 30-year-old man comes to the office because of excessive tearing of the left eye 3 months after repair of a deep laceration of the left medial canthus. Physical examination confirms epiphora of the left eye. A missed injury to the lacrimal drainage system is suspected. Which of the following is most appropriate to evaluate the suspected injury?
A) CT scan
B) Jones test
C) MRI
D) Nasal endoscopy
E) Schirmer test
A

B) Jones test

The most appropriate choice is the Jones test, which evaluates the drainage component of the lacrimal system. There are two steps in this test. The first step involves instilling two drops of a 2% fluorescein solution into the conjunctiva and observing the appearance or absence of this dye in the ipsilateral middle turbinate by means of a curved, cotton-tipped applicator. When dye is noted on the applicator within 3 minutes, the drainage system is intact. Therefore, if no obstruction is noted, the epiphora is caused by hypersecretion. If no dye passes through the nasolacrimal duct, then a secondary dye test is performed by force-injecting 1 mL of saline through the punctum. If this is done and the dye appears in the nose, then a functional or incomplete block of the nasolacrimal duct is present; if no dye appears, then a complete block is diagnosed. Differentiating a canalicular block from a nasolacrimal duct block is done by cannulating and injecting 1 mL of saline through a canaliculus and observing the passage of clear fluid out the other canaliculus. A No. 00 Bowman probe can also be passed through the canaliculus, and the distance to the obstruction can be measured. Typical distances are 8 mm to common canaliculus, 10 to 12 mm to the tear sac, and 16 mm to the upper end of the nasolacrimal duct.

CT scans are only useful in evaluating the nasal cavity. They cannot visualize the lacrimal drainage system unless the study is performed as a CT-dacryocystography, which is a specific subvariety of CT scanning, and is reserved as a confirmatory second-line test. Similarly, MRI can only identify abnormalities of the nasal cavity and not details of the drainage system.

Nasal endoscopy can only examine the nasal cavity for an abnormal location of the meatus under the middle turbinate. It can also determine if there is a physical obstruction of the lower end of the nasolacrimal duct at that location because of polyps or granulation tissue. This test could be indicated if the Jones test were to show partial obstruction.

The Schirmer test is considered a secretory test only and cannot be used to make any determination regarding tear excretion. The Schirmer test is most useful in diagnosing lacrimal hyposecretion. In the clinical scenario described, the epiphora appeared following the injury in the medial canthus; therefore, it is less likely to be related to hypersecretion. Yet a positive Jones test result (dye passes) in the presence of epiphora would clue the physician toward hypersecretion.

215
Q
An 18-year-old man comes to the emergency department 2 hours after being punched in the right eye during a fistfight. Physical examination shows swelling, diplopia, and a significant limitation of downward gaze in the affected eye. He is able to rotate the eye in all other directions. This finding is most consistent with dysfunction of which of the following extraocular muscles?
A) Inferior oblique
B) Inferior rectus
C) Superior oblique
D) Superior rectus
A

B) Inferior rectus

The patient described has a fracture of the orbital floor with entrapment or paralysis of the inferior rectus muscle. The inferior rectus muscle is innervated by cranial nerve III (oculomotor nerve) and is responsible for downward rotation of the eye.

The inferior oblique muscle is innervated by cranial nerve III and is responsible for upward and outward rotation of the eye. Entrapment or paralysis of the inferior oblique following orbital trauma limits downward gaze; it is difficult for patients to look down when walking down stairs. The superior oblique is innervated by the trochlear nerve (cranial nerve IV) and is responsible for downward and outward rotation of the eye. The superior rectus muscle is innervated by the oculomotor nerve and rotates the eye upward.

216
Q
A 23-year-old man comes to the office for evaluation of unilateral blepharoptosis. On examination, the excursion of the eyelid margin is measured from downgaze to upgaze while the eyebrow is manually fixed against the supraorbital rim. Which of the following would best approximate the normal excursion distance of levator function for this patient?
A) 2 to 6 mm
B) 7 to 11 mm
C) 12 to 16 mm
D) 17 to 21 mm
E) 21 to 25 mm
A

C) 12 to 16 mm

217
Q

A 46-year-old woman comes to the office for consultation about improving the appearance of her “saggy” upper eyelids. Physical examination shows moderate skin redundancy in both upper eyelids. The ciliary margin of the upper eyelid is located 1 mm below the superior limbus on the right and 3 mm below the superior limbus on the left. Levator excursion is 14 mm bilaterally. In addition to excision of excess skin bilaterally, which of the following is the most appropriate treatment?
A) Brow lift surgery on the left
B) Frontalis suspension in the left eye
C) Levator aponeurosis plication in the left eye
D) Orbicularis plication in the left eye

A

C) Levator aponeurosis plication in the left eye

The normal anatomical position of the ciliary margin of the upper eyelid is 1 mm below the upper corneoscleral junction in straight gaze. The upper corneoscleral junction is the landmark for ptosis measurement. Position of the eyelid in straight gaze along with the levator function determines the required degree of correction and the type of surgery. Patients with severe ptosis (3 mm or more) usually have poor levator function, therefore frontalis suspension is required. In patients with moderate function (6 to 10 mm), levator advancement and resection is required. For patients with excellent levator function (10 mm or more) and mild ptosis, aponeurotic surgery (plication) is appropriate. In levator advancement surgery, the eyelid will be elevated approximately 1 mm for every 3 mm of advancement.

The patient described has two distinct problems that require two different procedures. The upper eyelid skin redundancy can be addressed with a standard upper eyelid blepharoplasty with skin excision and, if needed, a slip of orbicularis muscle. The mild ptosis of her left upper eyelid will be accentuated after the blepharoplasty if left untreated. Since her levator function is excellent, her ptosis can be corrected with a simple plication of the distal levator aponeurosis. This can be accomplished through an upper blepharoplasty incision. In a Fasanella-Servat procedure, a portion of the posterior lamella of the eyelid is resected to improve mild eyelid ptosis by shortening the levator muscle. This procedure would not be used in a patient who requires a skin resection since it uses a posterior approach.

218
Q

A 60-year-old man comes to the office because he desires improvement in the appearance of his lower eyelid and upper cheek area. Physical examination shows a prominent tear trough. Which of the following is the anatomic basis of the tear trough?
A) Attachment of the orbital septum to the arcus marginalis
B) Cleft between the palpebral and orbital parts of the orbicularis oculi
C) Osteocutaneous ligament arising from the medial portion of the maxilla
D) Prominence of the orbital rim following descent of the malar fat pad
E) Triangular confluence of the origins of the orbicularis oculi, levator labii superioris alaeque nasi, and levator labii superioris

A

C) Osteocutaneous ligament arising from the medial portion of the maxilla

Recent articles have greatly improved our understanding of the anatomy of the lower eyelid, tear trough, pre-zygomatic space, and the ligaments in the orbital area.

The tear trough ligament is a true osteocutaneous ligament between the palpebral and orbital portions of the muscle. It extends inferolaterally from the medial canthus to approximately the mid-pupillary line, where is connects with the bilayered orbicularis retaining ligament. Recent anatomic dissection work has shown that this ligament is the anatomic basis of the tear trough deformity.

Fillers should be placed inferior to the tear trough ligament; placing them superiorly will only serve to emphasize lower eyelid fat, and will emphasize the tear trough deformity.

219
Q
A 32-year-old Korean woman undergoes bilateral upper lid blepharoplasty for creation of a supratarsal crease. A partial incision technique is used. Which of the following is the most likely complication of this procedure?
A) Asymmetry
B) Epicanthal webbing
C) Fold loss
D) Lagophthalmos
E) Suture extrusion
A

A) Asymmetry

Blepharoplasty is the most common facial cosmetic procedure performed on people of Asian descent. Unlike blepharoplasty in Caucasian faces, the goal of Asian blepharoplasty is to create a supratarsal fold. Asian eyelids are characterized by several key elements including absent or low lid crease, shorter tarsus, descending pre-aponeurotic fat, and minimal or absent connection between the levator aponeurosis and the upper lid dermis. The mainstay of surgical correction is creation of a permanent fixation point between the levator muscle and the supratarsal dermis and subdermal structures.

The most common complication after Asian blepharoplasty is asymmetry. It is important to remember that unlike Caucasian blepharoplasty, the motivation for Asian blepharoplasty is less frequently rejuvenation. Rather, Asian patients are typically younger and desire creation of a supratarsal fold or correction of a preexisting fold asymmetry. Asymmetry is a common preoperative finding and should be well documented and discussed with the patient before surgery. Small differences in positioning of the newly created crease can be very obvious to both patient and surgeon. When recognized immediately after surgery, early revision should be considered.

Other complications unique to Asian blepharoplasty include fold loss, suture extrusion, and epicanthal scarring. Lagophthalmos, which most often results from overresection of upper eyelid skin, is less frequent in Asian blepharoplasty because skin excision is typically more limited, particularly in a younger patient undergoing a partial incision technique. Fold loss may result from a technical error in securing the layers of dermis, epidermis, or levator aponeurosis, or from placing too few sutures. Revisional surgery would be required to correct this problem. Suture extrusion is not uncommon, given that permanent sutures are used for fixation. Meticulous placement of clear 7-0 nylon and trimming of suture ends will minimize the risk of this complication. If a suture becomes exposed in the first few months postoperatively, removal should be deferred until scarring is mature and fixation is more tenacious. Epicanthoplasty is commonly used is Asian patients with epicanthal folds undergoing blepharoplasty. Various techniques, including Y, W, and modified Z-plasties, have been reported. The epicanthal region is prone to hypertrophic scarring, particularly in Asians who have thick dermis.

220
Q

Which of the following is the function of secretions of the meibomian glands?
A) Coats cornea as inner layer of tear film
B) Lubricates eyelid skin
C) Prevents evaporation of tear film
D) Promotes control of infectious agents
E) Promotes dispersion of tear film

A

C) Prevents evaporation of tear film

Tears are a trilaminar fluid. The precorneal layer is formed by mucin-secreting goblet cells in the conjunctiva. This inner layer of the tear film covers the cornea and promotes the dispersion of the overlying aqueous layer.

The lacrimal gland secretes the middle layer. This aqueous layer is made of water and proteins. This layer promotes osmotic regulation and the control of infectious agents.

The meibomian glands produce the outer lipid layer. This oil layer helps to prevent the evaporation of the tear film. As a result, dysfunction of the meibomian glands can lead to dry eyes.

221
Q
In a patient with facial proportions within the normal ranges, which of the following measurements best approximates intercanthal distance?
A) Eyebrow length
B) Nasal bone width
C) Orbital fissure width
D) Stomion-to-menton distance
E) Subnasale-to-stomion distance
A

C) Orbital fissure width

Intercanthal distance most closely approximates orbital fissure width. Normal facial values are often described as proportions rather than absolute numbers. Many texts describe normal intercanthal distance as between 30 and 35 mm, but some studies have shown intercanthal distances of up to 40 mm in healthy cohorts. Thus, using the facial features as referents can be helpful. The face is often divided into fifths for analysis of width, and the intercanthal distance represents one fifth, as does the orbital fissure width. The nasal bone width is narrower than the intercanthal distance, and the eyebrow length extends lateral to the lateral canthus, representing greater than a fifth of the facial width. Subnasale, stomion, and menton distances are most often used to calculate facial height proportions. Although there is no reason why, theoretically, a measurement of facial height could not correspond to a measurement of facial width, these values do not.

222
Q
A 65-year-old woman comes to the office for follow-up 6 days after undergoing bilateral upper eyelid blepharoplasty and repair of the right levator aponeurosis. Preoperatively, the patient had bilateral levator excursion of 13 mm and 4 mm of ptosis of the right eyelid. No ptosis of the left eyelid was noted. Physical examination today shows 2 mm of ptosis of the left upper eyelid. The right upper eyelid is well positioned. Which of the following is the most likely explanation for these findings?
A) Hering law
B) Horner syndrome
C) Müller maneuver
D) Todd paresis
E) von Graefe sign
A

Hering law describes equal innervation to the eyelids in that the signal to the levator is the same despite the potential need for each eyelid to work independently. In the scenario described, the patient had obvious ptosis of the right eyelid, and the signal to raise the eyelids was strong. When the right ptosis was corrected, the signal to raise the eyelids decreased, and the more mild ptosis of the left eyelid was uncovered. To help avoid this problem, a Hering test or a patch test can be performed. The Hering test is performed by elevating the ptotic eyelid and observing whether the other eyelid becomes ptotic. A patch test is when the ptotic eyelid is covered for a period of time (usually 15 minutes) and then observed for whether the non-ptotic eye becomes ptotic. The key to both tests is to decrease the excessive signal to raise the eyelids.

Horner syndrome includes ptosis of the eyelid, constriction of the pupil, and decreased sweating due to disease in the sympathetic system. This can be due to a tumor, congenital or iatrogenic.

von Graefe sign is lagophthalmos in downgaze. This is related to Graves disease. Müller maneuver is the reverse of the Valsalva maneuver. After a forced expiration, an attempt at inspiration is made with closed mouth and nose, thereby creating negative pressure in the chest and lungs. This maneuver is used to find weakened areas of the airway.

Todd paresis is focal weakness following a seizure. This can affect eye position.

223
Q
A 48-year-old woman comes to the office because of pain and tearing of the right eye 1 week after undergoing upper eyelid blepharoplasty. Fluorescein stain test result is positive for corneal erosion. On physical examination, absence of which of the following is most likely to put this patient at risk for corneal ulceration?
A) Accommodation reflex
B) Bell phenomenon
C) Ocular convergence
D) Oculocardiac bradycardia
E) Pupillary light response
A

B) Bell phenomenon

Transient lagophthalmos during sleep is not uncommon following blepharoplasty. During the first few weeks of recovery, it is important to protect the eyes with lubricating drops and ointment. Bell phenomenon, an upward and outer movement of the eye when the eye is closed, is a protective mechanism which keeps the cornea protected behind the upper eyelid. Bell phenomenon is absent in 10 to 15% of the population. Accommodation reflex, oculocardiac reflex, ocular convergence, and pupillary light response do not place the cornea at risk after blepharoplasty.

224
Q

A 45-year-old woman comes to the office because of puffiness of both lower eyelids. Physical examination shows little lower lid skin excess, but prominent eyelid fat pads. Lid tone, snap back, and distraction test results show no abnormalities. Lid-cheek junction is smooth. A retroseptal transconjunctival approach of the lower lid is planned. Regarding the operative approach, which of the following statements is most accurate?
A) Fat pad reduction without violating the orbital septum is performed
B) Lower eyelid skin resurfacing with laser or chemical peel should not be performed concurrently with a retroseptal approach
C) The incision should be placed 1 to 2 mm below the tarsal border
D) The inferior oblique muscle will be noted between the central and lateral fat pad
E) The plane of dissection is deep to the orbicularis, but superficial to the orbital septum

A

A) Fat pad reduction without violating the orbital septum is performed

The incision for a retroseptal approach is usually placed 4 to 5 mm below the tarsal border, or about 8 mm for the lid margin. The plan of dissection for a retroseptal approach is, by definition, deep to both the orbicularis muscle and septum. Because the fat pads are retroseptal, modification does not require entry through the septum when a retroseptal approach is used.

The preseptal approach is typically chosen for modification of the lid-cheek junction, and facilitates fat pad redistribution and access to the midface. The retroseptal approach is used for reduction of fat pads only. The inferior oblique muscle is located between the central and medial compartments of fat.

Multiple authors have shown that is it indeed safe to perform skin resurfacing with either chemical peel or laser simultaneously with a transconjunctival blepharoplasty.

225
Q
A 50-year-old woman undergoes upper and lower eyelid blepharoplasty with local anesthesia and intravenous sedation. The procedure begins with no patient discomfort; however, the patient reports marked pain once removal of the lower lateral fat pad is initiated. Which of the following nerves is the source of pain in this patient?
A) Facial nerve
B) Infraorbital nerve
C) Infratrochlear nerve
D) Lacrimal nerve
E) Zygomaticofacial nerve
A

E) Zygomaticofacial nerve

The zygomaticofacial nerve provides sensory innervation to the lateral fat pad of the lower eyelid.

Branches of the trigeminal nerve provide sensation to the face. The infraorbital nerve, the second branch of the trigeminal nerve, supplies innervation to the lower eyelid, cheek, and upper lip. The lateral palpebral branch of the lacrimal nerve, a branch of the infraorbital nerve, supplies sensory innervation to the superior lateral portion of the upper eyelid. The infratrochlear nerve provides sensory innervation to the medial aspect of the upper and lower eyelid. The lacrimal nerve provides sensation to the upper eyelid through the first branch of the trigeminal nerve. The facial nerve is a motor nerve to the face and is not responsible for sensation in the face.

226
Q
A 50-year-old woman comes to the office for consultation because of upper eyelid dermatochalasis and lower eyelid bags. History includes hypothyroidism and type 2 diabetes mellitus. She underwent laser eye (Lasik) surgery 1 year ago. Medications include estrogen and thyroid hormone replacements as well as metformin. Upper and lower eyelid blepharoplasty is planned. This patient is at greatest risk for which of the following postoperative complications?
A) Bleeding
B) Blindness
C) Chemosis
D) Dry eye syndrome
Eyelid malposition
A

D) Dry eye syndrome

Dry eye syndrome (dysfunctional tear syndrome) is a problem of tear deficiency and eye discomfort that may result in damage to the cornea. Combining upper and lower eyelid blepharoplasty relative to staged upper and lower eyelid surgery presents a greater risk for dry eye syndrome due to orbicularis oculi dysfunction after surgery. Women on hormone replacement therapy also have a higher risk for developing dry eye syndrome after blepharoplasty. Eye lubrication before and after surgery needs to be strongly considered. Patients with prior laser vision correction should wait at least 6 months before pursuing blepharoplasty because of the effects on corneal sensation and tear production. In this case, the patient’s Lasik procedure 1 year prior would not impair her surgical outcome with upper and lower eyelid blepharoplasty.

Chemosis is characterized by conjunctival swelling and irritation after blepharoplasty surgery, requiring lubrication after surgery. Some surgeons advocate treatment of this condition with steroid drops.

Eyelid malposition is more likely in patients with a negative vector, defined as those with a maxilla that does not project beyond the orbital rim. It is also common in patients who have poor eyelid tone, diagnosed by snap test or evident as ectropion.

Major adverse postoperative events such as bleeding and loss of vision are rare. Bleeding is more risky with uncontrolled hypertension.

Smoking history, diabetes, and hypothyroidism are not directly associated with specific complications of blepharoplasty

227
Q
A 57-year-old woman comes to the office because she is dissatisfied with the appearance of her eyes. She says they appear "small" and "tired." Physical examination shows dermatochalasis of the upper eyelids, 2 mm of eyelid ptosis, deep transverse rhytides of the forehead, and fine periorbital rhytides. She elevates her eyebrows 3 mm when she opens her eyelids. A skin-only blepharoplasty with formal eyelid ptosis repair is planned. After the procedure, which of the following clinical findings is most likely in this patient?
A) Blepharospasm
B) Brow ptosis
C) Decreased pretarsal show
D) Decreased volumetric convexity
E) Eyelid retraction
A

B) Brow ptosis

A patient who undergoes blepharoplasty and ptosis repair in the context of a compensated brow ptosis is likely to experience worsened brow ptosis after the procedure. Evaluation of the blepharoplasty patient requires careful examination of the entire upper third of the face. Patients may have, in addition to excess upper eyelid skin, an eyelid ptosis. In addition to identifying the ptosis, it is important to recognize compensated brow ptosis. A compensated eyelid ptosis occurs when the patient uses the frontalis muscles to raise the eyebrows, which results in a functional improvement in visual fields. This is most easily identified by having the patient close her eyes, and evaluate the automatic raising of the eyebrow on eyelid opening. In this case, the change in the position of the brow on downward gaze and on frontal gaze indicates a compensated brow ptosis.

After ptosis repair and blepharoplasty, brow ptosis can become more manifest as the need for compensation decreases.

228
Q
A 48-year-old man comes to the office for consultation because he is dissatisfied with the appearance of his puffy, swollen eyelids and dry eyes. Physical examination shows mild bilateral proptosis and injected conjunctivae. Visual acuity and visual field testing are both normal. Extraocular motion testing reveals lid lag. Which of the following is the most likely cause of this patient?s condition?
A ) Allergic conjunctivitis
B ) Myasthenia gravis
C ) Optic neuritis
D ) Sjögren syndrome
E ) Thyroid ophthalmopathy
A

E ) Thyroid ophthalmopathy

The most appropriate answer is thyroid eye disease, or thyroid ophthalmopathy. This is the most common cause of proptosis and diplopia in adults. It affects women approximately 4 to 6 times more frequently than men. Puffy, swollen eyelids, injected conjunctivae, eyelid lag, and proptosis are common in thyroid ophthalmopathy.

Optic neuritis is typically associated with visual loss, which is not a symptom in this scenario. Similarly, myasthenia gravis is associated with eyelid ptosis, worsening toward the end of the day—which is not a symptom in this scenario. Although both allergic conjunctivitis and Sjögren syndrome could cause conjunctival injection, neither would be associated with proptosis or eyelid lag.

Vision loss, due to compression of the optic nerve, can occur in the most severe cases. Fortunately, this

229
Q
A 36-year-old woman is scheduled for lower blepharoplasty and mid face rhytidectomy using a transconjunctival approach. Which of the following structures is released to access the mid face for suborbicular muscle of the eye fat redraping?
A ) Capsulopalpebral fascia
B ) Lockwood ligament
C ) Orbital septum
D ) Orbitomalar ligament
E ) Parotid masseteric fascia
A

D ) Orbitomalar ligament

The orbitomalar ligament attaches the orbicular muscle of the eye to the orbital rim. It separates the lower eyelid from the mid face. Release of this structure is required to obtain access to the mid face when approaching it from the lower eyelid.

The capsulopalpebral fascia inserts on the inferior border of the tarsus. It makes up the anterior superior portion of the lower eyelid retractors distal to the Lockwood ligament. The capsulopalpebral fascia is divided during the transconjunctival incision. This affords access to the lower eyelid for the blepharoplasty procedure.

The Lockwood ligament is a fascial thickening that supports the globe. It surrounds the inferior rectus and inferior oblique muscles and fuses with the capsulopalpebral fascia. It is analogous to the Whitnall ligament in the upper eyelid. The orbital septum acts to contain the orbital contents. It attaches inferiorly to the periosteum at the arcus marginalis and superiorly to the eyelid margin. A transconjunctival blepharoplasty can be performed by a pre- or postseptal approach. The parotid masseteric fascia is within the mid face. It is distal to the orbitomalar ligament when approached from the eyelid. It does not need to be released to access the mid face.

230
Q
A 48-year-old woman is brought to the emergency department after sustaining facial injuries in a motor vehicle collision. Physical examination shows a 4-cm laceration of the left upper eyelid. The levator palpebrae superioris is transected just superior to the tarsus, exposing underlying structures. Which of the following intact structures is now exposed?
A ) Lacrimal sac
B ) Müller muscle
C ) Orbital septum
D ) Preaponeurotic fat
E ) Retro-orbicularis oculi fat
A

B ) Müller muscle

Knowledge of periorbital anatomy is critical for plastic surgeons who perform eyelid procedures, both cosmetic and reconstructive. The levator complex originates at the orbital apex at the lesser wing of the sphenoid and travels horizontally until it reaches the Whitnall ligaments, where it changes to a more vertical direction before its aponeurosis inserts on the tarsus, orbital septum, and skin. Above the level of the tarsus, the orbital septum lies anterior to the levator, and preaponeurotic fat lies posterior to the orbital septum.

Retro-orbicularis oculi fat lies anterior to the septum and posterior to the orbicularis oculi.

Müller muscle inserts directly on the tarsus and lies just posterior to the levator directly superior to the tarsus.